Учебное пособие по математике

Раздел Математика
Класс -
Тип Другие методич. материалы
Автор
Дата
Формат docx
Изображения Есть
For-Teacher.ru - все для учителя
Поделитесь с коллегами:
Учебное пособие по математикеУчебное пособие по математикеУчебное пособие по математикеУчебное пособие по математикеУчебное пособие по математикеУчебное пособие по математикеУчебное пособие по математикеУчебное пособие по математикеУчебное пособие по математикеУчебное пособие по математикеУчебное пособие по математике

ГОСУДАРСТВЕННОЕ ПРОФЕССИОНАЛЬНОЕ ОБРАЗОВАТЕЛЬНОЕ УЧРЕЖДЕНИЕ

«ПРОКОПЬЕВСКИЙ ЭЛЕКТРОМАШИНОСТРОИТЕЛЬНЫЙ ТЕХНИКУМ»



Учебное пособие по математике















Данное пособие направлено на овладение студентом общим подходом к поиску способа решения разного вида задач как одним из общеучебных умений.

Результаты освоения программы практикума

Каждый имеет возможность

1) относительно текстовых задач научиться:

- «видеть» задачные ситуации в окружающей жизни и формулировать вопросы к ним;

- представлять задачные ситуации в виде устного текста, рисунка, модели, схемы, математической записи;

- осуществлять переход от одной формы представления к другой;

- отличать текст задачи от других видов текстов;

- выявлять структуру задачи;

- по условию подбирать, составлять вопросы;

- составлять задачи по определённой теме;

- представлять жизненную ситуацию, описанную в задаче;

- разбивать текст задачи на смысловые части и анализировать каждую часть;

- переформулировать текст задачи;

- составлять план решения задачи;

- фиксировать решение задачи;

- проверять правильность решения задачи;

- составлять задания по решённой задаче.

2) Относительно коммуникативных задач научиться:

- вести диалог;

- самостоятельно составлять рассказ на заданную тему по картинке;

- по схеме обосновать логику своего изложения;

- формулировать вопрос на понимание, уточнение;

- высказывать собственное суждение, мнение;

- слушать и понимать другого;

- организовывать работу малых групп и быть их участником.

3) Относительно информационных задач научиться:

- ориентироваться в книге;

- искать информацию в словарях, энциклопедиях, справочниках;

- пользоваться изученной математической терминологией;

- отделять известное от неизвестного, главное от второстепенного;

- формулировать познавательные вопросы и задания;

-формулировать собственные выводы.

г. Прокопьевск 2016 год

СОДЕРЖАНИЕ



Введение.................................................................................................................5-7

Раздел1. Практическая геометрия...................................................................8-11

Задачи с практическим содержанием:

Параллельность......................................................................................................12

Четырехугольники.................................................................................................13

Вписанные и описанные многоугольники..........................................................14

Движение................................................................................................................15

Подобие...................................................................................................................16

Тригонометрические функции острого угла.......................................................17-18

Площадь многоугольников...................................................................................19-20

Площадь круга и его частей..................................................................................21

Координаты и векторы..........................................................................................22

Тригонометрия.......................................................................................................23-24

Элементы стереометрии........................................................................................25-32

Раздел 2. Математическая логика.....................................................................33-35

Метод логических квадратов................................................................................36-38

Логическая арифметика.........................................................................................39-40

Задачи для самостоятельного решения на логику...............................................41

Раздел 3. Решение нестандартных текстовых задач. "Исключение

невозможных значений, подбор ответа, рекомендация по решению нестандартных задач, задачи из "числа".........................................................42-44

Арифметический метод..........................................................................................45-47

Алгебраический метод............................................................................................48

Комбинированный метод.....................................................................................49-50

Примеры решения задач.......................................................................................51-53

Раздел 4. Задачи на оптимизацию....................................................................54-56

Памятка по решению задач на оптимизацию......................................................57

Примеры решения задач на оптимизацию..........................................................58-66

Задачи для самостоятельного решения................................................................67

Тестовые задания....................................................................................................68

Раздел 5. Комбинаторика.....................................................................................69-71

Формула включений и исключений.......................................................................72

Определение факториала.........................................................................................73

Задачи по теме: Размещение, перестановки, сочетания.......................................74-75

Подбор и решение задач на основные формулы комбинаторики и Бином

Ньютона.....................................................................................................................76-77

Задачи для самостоятельного решения...................................................................78-82

Раздел 6. Элементы теории вероятностей и математической статистики...83-85

Как решать задачи на вероятность..........................................................................86-88

Задачи для самостоятельного решения...................................................................89-93

Заключение.................................................................................................................94-95

Список литературы....................................................................................................96-98









Введение

В обучении математике задачам всегда отводилась достаточно большая, если не решающая, роль. Сейчас всё большее распространение получает прогрессивный метод обучения через задачи как реализация системы проблемного обучения. Задачи становятся не только и не столько целью, сколько средством обучения.
Исторически сложилось, что на ранних этапах развития математики решение задач было целью обучения. Ученик должен был заучить образцы и затем подводить под эти образцы решения задач. В основном решались типовые, стандартные задачи, принадлежащие классам алгоритмически разрешимых задач, т.е. таких, для которых существует общий метод (алгоритм) решения. Многообразные ситуации, возникающие на математическом и нематематическом материале, приводят как к стандартным, так и нестандартным задачам, алгоритм решения которых либо неизвестен, либо не существует.
В последние десятилетия постепенное изменение целей обучения математике приводит к необходимости учить детей решению не только стандартных, но и нестандартных задач, которые нельзя отнести к классу алгоритмически разрешимых. Именно по отношению к нестандартной задаче возникает необходимость в вариативном поиске решения. "Задача предполагает необходимость сознательного поиска соответствующего средства для достижения ясно видимой, но непосредственно не доступной цели. Решение задач означает нахождение этого средства". Роль задач в обучении математике невозможно переоценить. Через задачу естественно ввести проблемную ситуацию. Разрешив систему специально подобранных задач, ученик знакомится с существенными элементами новых алгоритмов, овладевает новыми техническими элементами. Применять математические знания в жизненных ситуациях учат соответствующие практические задачи. Итак, как видно из приведённого выше обзора мнений различных специалистов в области образования и обучения математике, задача является основным звеном внутри процесса обучения, а тем более такого, как проблемное и развивающее.

Цели обучения математике определяется ее ролью в развитии общества в целом и формировании личности каждого человека. Для жизни в современном обществе важным является формирование математического стиля мышления, проявляющегося в определенных умственных навыках. Наряду с решением основной задачи (обеспечение прочного и сознательного овладения учащимся системой математических знаний и умений, необходимых в повседневной жизни и трудовой деятельности, достаточных для изучения смежных дисциплин и продолжения образования) факультативный курс предусматривает формирование у учащихся устойчивого интереса к предмету, выявление и развитие их математических способностей, ориентацию на профессии, существенным образом связанные с математикой, подготовку к обучению в ВУЗе.

Изменения, вносимые Федеральным компонентом государственного стандарта общего образования в требования к математической подготовке учащихся, должны найти отражение в технологии и содержании контроля математической подготовки. Проверка усвоения содержания курса математики предполагает включение в содержание контрольных работ заданий из всех 4 категорий познавательной области:

- знание/понимание: владение термином; владение различными эквивалентными представлениями (например, числа); распознавание (на основе определений, известных свойств, сформированных представлений); использование различных математических языков (символического, графического, вербального), переход от одного языка к другому; интерпретация;

- алгоритм: использование формулы как алгоритма вычислений; применение основных правил действий с числами, алгебраическими выражениями; решение основных типов уравнений, неравенств, систем;

- решение задачи: задания с математическим контекстом, при решении которых требуется применение (актуализация) системы знаний, преобразование связей между известными фактами, включение известных понятий, приемов и способов решения в новые связи и отношения; распознавание стандартной задачи в измененной формулировке;

- практическое применение: задание, соответствующее одной из первых трех категорий данного списка, формулировка которых содержит практический контекст, знакомый учащимся или близкий их жизненному опыту.

В результате учащиеся должны уметь:

  • быстро и свободно выполнять действия с числовыми и алгебраическими выражениями;

  • хорошо владеть техникой тождественных преобразований целых и дробных рациональных выражений и выражений , содержащих степень и корень n-ой степени;

  • усвоить основные приемы решения уравнений, неравенств, систем уравнений и неравенств;

  • решать текстовые задачи методом уравнений;

  • решать иррациональные, тригонометрические уравнения и неравенства, доказывать неравенства, уравнения и неравенства с модулем;

  • решать нестандартные уравнения и неравенства;

  • уметь решать уравнения, содержащие параметр;

  • строить графики элементарных функций и проводить преобразования графиков, использовать изученные методы;

  • применять метод математической индукции и бином Ньютона;

  • изображать на рисунках и чертежах пространственные геометрические фигуры и их комбинации;

  • усвоить определенный набор приемов решения геометрических задач и уметь их применять в задачах на вычисление, доказательство и построение;

  • применять основные методы геометрии (проектирование, преобразований, векторный, координатный) к решению задач.

  • расширение представления об уравнениях, неравенствах, системах и методах их решения;

  • развитие логической культуры, составляющей существенный компонент культуры мышления, рассматриваемой в рамках общей культуры;

  • овладение общими приемами организации действий: планирование, осуществление плана, анализ и представление результатов действий;

  • развитие внутренней мотивации и фактора поисковой активности в предметной деятельности, формирование устойчивого и осознанного интереса к ней.

Способствует развитию у учащихся следующих компетенций:

Предметные:

  • умение проводить логически грамотные преобразования выражений и эквивалентные преобразования алгебраических задач (уравнений, неравенств, систем, совокупностей);

  • умение использовать основные методы при решении алгебраических задач с различными классами функций;

  • умение понимать и правильно интерпретировать алгебраические задачи, умение применять изученные методы исследования и решения алгебраических задач.

Общеинтеллектуальные:

  • умение анализировать различные задачи и ситуации, выделять главное;

  • умение логически обосновывать свои суждения;

  • умение конструктивно подходить к предлагаемым задачам;

  • умение планировать свою деятельность, проверять и оценивать её результаты.

Общекультурные:

  • восприятие математики как развивающейся фундаментальной науки, являющейся неотъемлемой составляющей науки, цивилизации, общечеловеческой культуры во взаимосвязи и взаимодействии с другими областями мировой культуры.

Учебное пособие по математике

РАЗДЕЛ 1. Практическая геометрия

Учебное пособие по математике

Геометрия - один из важнейших компонентов математического образования, она необходима для приобретения конкретных знаний о пространстве и практически значимых умений, формирования языка описания объектов окружающего мира, развития пространственного воображения и интуиции, математической культуры и эстетического воспитания учащихся. Изучение геометрии вносит вклад в развитие логического мышления и формирование понятия доказательства.

Ценностные ориентиры содержания геометрии

Математическое образование играет важную роль, как в практической, так и в духовной жизни общества. Практическая сторона математического образования связана с формированием способов деятельности, духовная - с интеллектуальным развитием человека, формированием характера и общей культуры.

Практическая полезность геометрии обусловлена тем, что ее предметом являются фундаментальные структуры реального мира: пространственные формы и количественные отношения - от простейших, усваиваемых в непосредственном опыте, до достаточно сложных, необходимых для развития научных и технологических идей. Без конкретных математических знаний затруднено понимание принципов устройства и использования современной техники, восприятие и интерпретация разнообразной социальной, экономической, политической информации, малоэффективна повседневная практическая деятельность. Каждому человеку в своей жизни приходится выполнять достаточно сложные расчеты, находить в справочниках нужные формулы и применять их, владеть практическими приемами геометрических измерений и построений, читать информацию, представленную в виду таблиц, диаграмм, графиков, понимать вероятностный характер случайных событий, составлять несложные алгоритмы и др.

Без базовой математической подготовки невозможно стать образованным современным человеком. Математика служит опорным предметом для изучения смежных дисциплин. В после школьной жизни реальной необходимостью в наши дни является непрерывное образование, что требует полноценной базовой общеобразовательной подготовки, в том числе и математической. И наконец, все больше специальностей, где необходим высокий уровень образования, связано с непосредственным применением математики (экономика, бизнес, финансы, физика, химия, техника, информатика, биология, психология и др.). Таким образом, расширяется кругстудентов, для которых математика становится значимым предметом.

Для жизни в современном обществе важным является формирование математического стиля мышления, проявляющегося в определенных умственных навыках. В процессе математической деятельности в арсенал приемов и методов человеческого мышления естественным образом включаются индукция и дедукция, обобщение и конкретизация, анализ и синтез, классификация и систематизация, абстрагирование и аналогия. Объекты математических умозаключений и правила их конструирования вскрывают механизм логических построений, вырабатывают умения формулировать, обосновывать и доказывать суждения, тем самым развивают логическое мышление. Ведущая роль принадлежит математике в формировании алгоритмического мышления и воспитании умений действовать по заданному алгоритму и конструировать новые. В ходе решения задач - основной учебной деятельности на уроках математики - развиваются творческая и прикладная стороны мышления.

Обучение математике дает возможность развивать у студентов точную, экономную и информативную речь, умение отбирать наиболее подходящие языковые (в частности, символические, графические) средства.

Математическое образование вносит свой вклад в формирование общей культуры человека. Необходимым компонентом общей культуры в современном толковании является общее знакомство с методами познания действительности, представление о предмете и методе математики, его отличия от методов естественных и гуманитарных наук, об особенностях применения математики для решения научных и прикладных задач.

Изучение геометрии способствует эстетическому воспитанию человека, пониманию красоты и изящества математических рассуждений, восприятию геометрических форм, усвоению идеи симметрии.

В результате изучения геометрии студент должен:

знать/понимать

  • значение математической науки для решения задач, возникающих в теории и практике; широту и ограниченность применения математических методов к анализу и исследованию процессов и явлений в природе и обществе;

  • возможности геометрического языка как средства описания свойств реальных предметов и их взаимного расположения;

  • универсальный характер законов логики математических рассуждений, их применимость в различных областях человеческой деятельности;

  • различие требований, предъявляемых к доказательствам в математике, естественных, социально-экономических и гуманитарных науках, на практике;

  • роль аксиоматики в математике; возможность построения математических теорий на аксиоматической основе; значение аксиоматики для других областей знания и для практики;

Уметь:

  • соотносить плоские геометрические фигуры и трехмерные объекты с их описаниями, чертежами, изображениями; различать и анализировать взаимное расположение фигур;

  • изображать геометрические фигуры и тела, выполнять чертеж по условию задачи;

  • решать геометрические задачи, опираясь на изученные свойства планиметрических и стереометрических фигур и отношений между ними, применяя алгебраический и тригонометрический аппарат;

  • проводить доказательные рассуждения при решении задач, доказывать основные теоремы курса;

  • вычислять линейные элементы и углы в пространственных конфигурациях, объемы и площади поверхностей пространственных тел и их простейших комбинаций;

  • применять координатно-векторный метод для вычисления отношений, расстояний и углов;

  • строить сечения многогранников и изображать сечения тел вращения.

Использовать приобретенные знания и умения в практической деятельности и повседневной жизни для

  • исследования (моделирования) несложных практических ситуаций на основе изученных формул и свойств фигур;

  • вычисления длин, площадей и объемов реальных объектов при решении практических задач, используя при необходимости справочники и вычислительные устройства.

Общеучебные умения, навыки и способы деятельности

В ходе изучения математики студенты продолжают овладение разнообразными способами деятельности, приобретают и совершенствуют опыт:

  • проведения доказательных рассуждений, логического обоснования выводов, использования различных языков математики для иллюстрации, интерпретации, аргументации и доказательства;

  • решения широкого класса задач из различных разделов курса, поисковой и творческой деятельности при решении задач повышенной сложности и нетиповых задач;

  • планирования и осуществления алгоритмической деятельности: выполнения и самостоятельного составления алгоритмических предписаний и инструкций на математическом материале;

  • использования и самостоятельного составления формул на основе обобщения частных случаев и результатов эксперимента;

  • выполнения расчетов практического характера;

  • построения и исследования математических моделей для описания и решения прикладных задач, задач из смежных дисциплин и реальной жизни;

  • проверки и оценки результатов своей работы, соотнесения их с поставленной задачей, с личным жизненным опытом;

  • самостоятельной работы с источниками информации, анализа, обобщения и систематизации полученной информации, интегрирования ее в личный опыт.


ЗАДАЧИ С ПРАКТИЧЕСКИМ СОДЕРЖАНИЕМ

Параллельность


1. Постройте параллельные прямые с помощью линейки и: а) чертежного угольника; б) циркуля.

2. Через данную точку (не принадлежащую данной прямой) проведите с помощью транспортира и линейки прямую, параллельную данной прямой.

3. Начертите две прямые, параллельные верхнему краю тетради.

4. На плане города улицы обозначенные как AB и CD, параллельны (рис. 35). Улица EF составляет с улицами AB и AC углы соответственно =43 и=65. Найдите углы, которые образуют между собой улицы AC и AB, AC и CD.

Учебное пособие по математике

5. Как практически проверить, параллельны ли две прямые: а) изображенные в тетради; б) провешенные на местности?

6. С помощью одной линейки постройте сумму углов A и B треугольника ABC.

7. На рисунке 36 изображен прибор, который называется эклиметр. Он используется для измерения углов в вертикальной плоскости при проведении работ на местности. Объясните, что измеряется с помощью него и как он устроен.

8. На рисунке 37 показано, как с помощью чертежного угольника построены две перпендикулярные прямые AB и BC. Объясните это построение.

Учебное пособие по математике

9. Найдите угол, образованный линиями насечек у напильника, изображенного на рисунке 38.

10. По одну сторону от шоссе расположены два дачных участка. Нужно проложить дорогу, параллельную шоссе, таким образом, чтобы сумма расстояний от участков до нее была наименьшей.





Четырехугольники


11. На рисунке 39 изображено устройство, которое называется «параллельные линейки» и используется для построения параллельных прямых. Объясните, как им пользуются.

Учебное пособие по математике

12. Объясните по рисунку 40, как находится расстояние между недоступными объектами X и Y, которые находятся, например, на противоположном от наблюдателя берегу реки, MN - искомое расстояние.

13. В старинных паровозах на колесах закреплялся специальный стержень (на рисунке 41 ST), равный расстоянию между центрами соответствующих окружностей, который передавал движение от первого колеса ко второму, равному ему колесу. Объясните, как должен быть расположен стержень относительно линии центров (прямая, соединяющая центры двух окружностей).

14. В прямоугольной пластине нужно просверлить круглое отверстие на равном расстоянии от вершин. Как найти центр отверстия?

15. Мастеру заказали изготовить ставни, чтобы закрыть прямоугольные окна на даче. Какие размеры он должен снять с каждого окна?

16. Как, имея двустороннюю линейку, построить ромб?

17. Как, имея двустороннюю линейку, данный угол: а) разделить пополам; б) удвоить?

18. Как проверить, что салфетка имеет форму квадрата? Достаточно ли перегнуть ее по: а) одной диагонали; б) двум диагоналям?

19. На одной прямой на равном расстоянии друг от друга стоят три телеграфных столба. Крайние находятся от дороги на расстояниях 18 м и 48 м. Сделайте соответствующий рисунок и найдите расстояние, на котором находится от дороги средний столб.

20. Участок между двумя параллельными улицами имеет вид четырехугольника ABCD (AD||BC) AB=28 см, BC=35 см, AD=42 см, Учебное пособие по математикеB=112. Выберите масштаб и нарисуйте план участка. Найдите приблизительно периметр участка и количество досок шириной 10 см, которые нужно заготовить, чтобы обнести его плотным забором.








Вписанные и описанные многоугольники

21. В металлической пластине просверлены три круглых отверстия, их центры не принадлежат одной прямой. Где нужно просверлить четвертое отверстие, чтобы его центр находился на равном расстоянии от центров трех данных отверстий?

22. В треугольной пластине нужно просверлить круглое отверстие, чтобы оно находилось на равных расстояниях от ее сторон. Где должен быть центр отверстия?

23. В каком месте потолка нужно повесить люстру, чтобы все углы прямоугольной комнаты были одинаково освещены?

24. В каком месте ромбовидной поляны нужно встать, чтобы эхо одинаково отражалось от всех стен леса?

25. В окружность без указанного центра с помощью чертежного угольника впишите квадрат.

26. Из фанеры требуется выпилить круглую крышку для бочки. Какие измерения нужно сделать мастеру, имея в своем распоряжении одну рулетку?

27. Для того чтобы предупредить корабли об имеющихся на их пути трех мелях A, B, C (рис. 42), поставили два маяка M1 и M2, которые расположены на окружности, охватывающей опасный участок. Угол M1OM2 должен быть известен лоцманам, ведущим корабли. Как могут лоцманы, измеряя угол между направлениями на оба маяка, узнавать, находятся ли корабли вне зоны опасности?

Учебное пособие по математике

28. Ученику нужно провести трисекцию (разделить на три равные части) центрального угла на выпиленной круглой пластине. Он поступил следующим образом: провел хорду, соответствующую данному центральному углу, разделил ее на три равные части и точки деления соединил с центром пластины. Разделился ли при этом центральный угол на три равные части? Обоснуйте свой ответ.

29. Известно, что громоотвод защищает от молнии на расстоянии от его основания не более его удвоенной высоты. Где на дачном участке лучше всего разместить громоотвод, чтобы его высоту сделать наименьшей, если участок имеет форму: а) круга; б) прямоугольного треугольника; в) равностороннего треугольника; г) разностороннего треугольника; д) прямоугольника; е) квадрата.

30. Садовник разбил красивую круглую клумбу. На ней, в частности, посадил маргаритки, которые образовали стороны вписанного четырехугольника. У него остались еще цветы, и он продолжил стороны четырехугольника до их пересечения и посадил также цветы по биссектрисам углов, образованных продолжениями этих сторон. Ему показалось, что эти биссектрисы перпендикулярны и соответственно параллельны биссектрисам углов, образованных диагоналями того же четырехугольника. Верно ли предположение садовника?


Движение


31. Как, используя центральную симметрию, измерить расстояние между двумя объектами, между которыми находится, например, дом?

32. Как, используя центральную симметрию, измерить расстояние между двумя объектами M и N, если между ними два дома D1, D2 и невдалеке кустарник К (рис. 43).

Учебное пособие по математике

33. На участке прямоугольной формы находятся две дачи D1, D2 и колодец К (рис. 44). Как нужно поставить забор, чтобы участки дач были равны и колодец находился на их границе?

34. На участке имеется площадка. Как провести прямую изгородь, чтобы она разделила и участок, и площадку на две равные части, если: а) и участок, и площадка имеют прямоугольную форму (рис. 45,а); б) участок имеет форму параллелограмма, площадка - круга, и расположены они так, как показано на рисунке (рис. 45,б)?

Учебное пособие по математике

35. Между двумя пунктами P и Q: а) протекает река (рис. 46,а); б) протекают две реки (рис. 46,б). Где нужно построить переправу, чтобы соединить пункты самой короткой дорогой?

Учебное пособие по математике

36. Как восстановить садовый участок квадратной формы, если сохранились три столбика от ограды его периметра - два на противоположных сторонах и один - в центре?

37. Как восстановить участок квадратной формы, если от его ограды сохранились четыре столбика - по одному на каждой его стороне? Всегда ли это можно сделать?

38. В каком направлении нужно ударить бильярдный шар X (рис. 47), чтобы он, ударившись о стенку BC прямоугольного стола, попал в шар Y?

39. В каком направлении нужно ударить бильярдный шар X (рис. 47), чтобы он, последовательно ударившись о четыре стенки AB, BC , CD и ADпрямоугольного стола, попал в шар Y?

40. На клетчатой бумаге в вершинах клеток поставлены две точки O и P так, как показано на рисунке 48. Не проводя никаких линий, найдите точкуP1, в которую перейдет точка P при повороте вокруг точки O на угол, равный -90Учебное пособие по математике.

Учебное пособие по математике



Подобие



41. На рисунке 49 изображен прибор - дальнометр, где AL - линейка со шкалой, PQ - подвижная планка. Как можно найти с помощью этого прибора расстояние, например, AK, зная расстояние между объектами B и C?

42. По данному рисунку 50 объясните, как определяется высота H дерева. При этом используется шест, на рисунке он обозначен CD. Чему равна высота H, если h, S, a, b известны?

Учебное пособие по математике

43. Как определить высоту дерева по его тени в солнечный день?

44. Изображение дерева на фотопленке имеет высоту 15 мм. Найдите высоту дерева, если расстояния от объектива фотоаппарата до изображения и до дерева равны соответственно 50 мм и 60 м (рис. 51).

Учебное пособие по математике

45. На географической карте три населенных пункта расположены друг от друга на расстояниях 6 см, 5 см и 4,5 см. Наибольшее расстояние равно в действительности 15 км. Найдите масштаб карты и действительное наименьшее расстояние.

46. Какой должна быть ширина (x) прямоугольной рамки для фотографий, если известны три ее размера a, b, c, указанные на рисунке 52, чтобы прямоугольники рамки и фотографии были подобны?

47. На рисунке 53 изображен прибор, который называется «делительный циркуль». Объясните, как, пользуясь им, можно снять копии отдельных фрагментов планов, чертежей, деталей и т.п. Например, найдите расстояние BC, чтобы циркуль: а) увеличил оригинал в 2 раза; б) уменьшил оригинал в 3 раза. Ножка AE имеет длину 15 см.

Учебное пособие по математике

48. На рисунке 54 два человека M1 и M2 должны выйти к шоссе GH в одном месте. Вектор Учебное пособие по математике задает направление движения M1. Определите соответствующий вектор направления движения M2.

49. Нужно найти расстояние, на котором упадет от теннисной сетки поданный мяч, если ее высота равна 90 см, мяч подан с высоты 2,4 м от конца площадки, длина которой до сетки равна 12 м.

50. Объясните по рисунку 55, как измерить глубину обрыва, стоя на его краю и имея небольшую палку: A - уровень глаз стоящего человека, B - уровень обрыва и уровень глаз лежащего человека, CD и EF - длины частей палки, K - камень на дне.




Тригонометрические функции острого угла


51. Горная железная дорога поднимается на 1 м на каждые 30 м пути. Найдите угол подъема.

52. На каком расстоянии друг от друга следует копать ямки для посадки деревьев по склону холма, наклоненному к горизонту под углом , если расстояние между двумя деревьями на ровной горизонтальной поверхности равно a м?

53. Ширина каждой ступеньки (называется проступь) лестницы равна b см. Найдите высоту ступеньки, если угол подъема лестницы равен .

54. Используя теорему косинусов, определите расстояние между пунктами M и N, между которыми расположен пруд (рис. 56).

Учебное пособие по математике

55. Используя теорему синусов, предложите способ определения расстояния между двумя пунктами (R и S), которые находятся на разных берегах реки.

56. На рисунке 57 показано, как определяли расстояние между двумя недоступными объектами E и F, находящимися на другом берегу реки. Объясните предложенный способ определения EF.

57. Длина маятника равна l м, высота его подъема (от вертикального положения) при отклонении на угол Учебное пособие по математике равна h м. Найдите расстояние от конца маятника при данном отклонении от вертикальной прямой до его первоначального спокойного состояния.

58. На рисунке 58 угол AOB - угол места цели (это угол между горизонтом и прямой, соединяющей орудие с целью). Найдите его при стрельбе по цели, которая находится выше уровня орудия на 75 м, учитывая, что расстояние Учебное пособие по математике от орудия до цели по карте масштаба 1:10000 равно 37,5 см.

Учебное пособие по математике

59. На стрельбище спортсмены выстраиваются параллельно стрелковому стенду на расстоянии 500 м от него. Определите длину участка, который находится под обстрелом, если расстояние между первым и последним участниками соревнования равно 100 м и дальность полета пули равна 2,9 км.

60. К одной материальной точке M под углом 60 друг к другу приложены две силы P1=100 кг и P2=200 кг. Найдите величину равнодействующей Rи углы, которые она составляет с P1 и P2.




Площадь многоугольников


1. Найдите площадь участка, план которого приведен на рисунке 15. (Размеры даны в метрах.) Выразите площадь в: а) арах; б) гектарах.

Учебное пособие по математике

2. Площадь земельного участка, имеющего форму прямоугольника, равна 9 га, ширина равна 150 м. Найдите длину этого участка.

3. Пол в танцевальном зале имеет форму прямоугольника размером 8 м x 15 м. Его требуется покрыть паркетной плиткой. Сколько нужно подготовить квадратных плиток, размером 50 см x 50 см, если на обрезки и пригонку затрачивается 2% площади всех плиток.

4. Вычислите давление, которое оказывает прибор весом 7,65 т на на 1 см2 своего фундамента, имеющего форму равностороннего треугольника со стороной 3 м.

5. На рисунке 16 изображен поперечный профиль дороги. Вычислите его площадь, если ширина полотна дороги a = 7,5 м, h - стрела подъема полотна над насыпью составляет 2% ширины полотна, откосы наклонены к линии горизонта под углом 45Учебное пособие по математике и высота насыпи H = 1,5 м.

Учебное пособие по математике

6. Земельный участок треугольной формы, назовем его MNK, площади 5 га нужно разделить межой MX на две части так, чтобы площадь MXK составила 1,5 га. Найдите длину KX, если KN = 40 м.

7. Земельный участок имеет форму треугольника. Как разделить его на три равновеликих участка таким образом, чтобы в каждом было по одной стороне треугольника?

8. Найдите наиболее простой способ деления поля, имеющего форму параллелограмма на: а) две; б) четыре равновеликие части прямыми, выходящими из одной вершины.

9. Как через точку внутри квадратной поляны провести прямую тропинку так, чтобы она отсекла участок наименьшей площади.

10. На куске фанеры, имеющей форму параллелограмма сделали разметку, как показано на рисунке 17, где KL || AB и MN || BC, и выпилили четырехугольники AMPK и PLCN. Равновелики ли эти четырехугольники? Зависит ли равновеликость от выбора точки P на диагонали BD?

11. Найдите площадь поперечного сечения ромбовидных напильников, диагонали которых равны: а) 1,3 см и 36 мм; б) 2,1 мм и 2,7 см.

12. Какие измерения нужно произвести, чтобы найти площадь вырезанной из квадратного листа фанеры рамки, имеющей форму: а) восьмиугольника (рис. 18, а); б) греческого креста (рис. 18, б); в) восьмиугольника (рис. 18, в). (Каждая сторона квадрата разделена на три равные части.) Найдите площадь рамки, если взят единичный квадрат.

Учебное пособие по математике

13. Детская площадка квадратной формы огорожена забором, закрепленном с помощью четырех столбов, находящихся в вершинах квадрата. Как увеличить площадь в два раза, чтобы новая площадка тоже имела форму квадрата, и столбы остались по ее периметру.

14. Из листа цветной бумаги прямоугольной формы (рис. 19, AE || CF) вырезали два треугольника. Найдите процент оставшейся от листа площади, если DC= 20 см, AF = 2,2 см.

Учебное пособие по математике

15. Из листа фанеры прямоугольной формы размером 220 см x 160 см необходимо вырезать заготовки в виде равнобедренных трапеций с основаниями 20 см, 60 см и углом 45. Сколько заготовок получится из данной фанеры? Определите процент неизрасходованной площади.

16. Найдите площадь клумбы, имеющей форму трапеции, если одна из ее боковых сторон равна 8 м, угол 30 и известно, что вокруг нее можно описать окружность и в нее можно вписать окружность.

17. Пол кухни размера 3 м x 3 м нужно застелить линолиумом, состоящим из плиток формы: а) правильных восьмиугольников и квадратов; б) правильных шестиугольников. Сколько потребуется плиток, если их стороны равны 15 см?

18. Как разделить участок, имеющий форму выпуклого четырехугольника на две равновеликие части межой, проведенной через одну из его вершин?

19. Сколько потребуется краски, чтобы выкрасить с двух сторон железный щит в виде прямоугольного треугольника, если гипотенуза треугольника равна 5 м, разность катетов равна 3 м и на 5 см2 расходуется 1,8 г краски.

20. Внутри участка, имеющего форму правильного шестиугольника, решили посадить цветы в виде клумбы, также имеющей форму шестиугольника и образованной меньшими диагоналями внешнего шестиугольника. Найдите площадь клумбы, если площадь участка равна 8,4 м2.


Площадь круга и его частей


21. Дерево имеет в обхвате 1,2 м. Найдите площадь поперечного сечения в этом месте, имеющего приблизительно форму круга.

22. Часы, установленные на высотном здании Московского государственного университета имеют диаметр около 8,8 м. Найдите площадь, которую занимает циферблат этих часов. Сравните ее с площадью вашего кабинета.

23. Найдите площадь бумажного змея, чертеж которого приведен на рисунке 20 в масштабе 1 клетка = 0,1 м.

Учебное пособие по математике

24. Из медного квадратного листа вырезали круг наибольшего диаметра. Найдите площадь листа, если площадь круга равна 68,68 см2.

25. Сколько нужно семян, чтобы засеять круглую клумбу диаметром 3,2 м, если на 1 см2 идет 0,0004 г?

26. Сколько нужно песка, чтобы окружить круглую клумбу прилегающей к ней дорожкой шириной 0,5 м, если на 1 м2 требуется 0,8 дм3 песка и наибольший диаметр равен 18 м?

27. В будильнике минутная стрелка длиннее часовой на 8 мм. Найдите длину этих стрелок, если площадь кольца, заключенного между окружностями, которые они описывают, равна 13,5 см2.

28. На рисунке 21 изображена мишень. В каком отношении находятся площади ее наименьшего круга и круговых колец, если ширина каждого кольца равна радиусу внутреннего круга.

29. Найдите площади сечений деталей, заштрихованных на рисунке 22, приняв стороны равностороннего треугольника и квадрата за единицу. В случае в) сторона квадрата разделена на три равные части, в случае д) в отношении 1 : 2 : 1.

30. Найдите площади деталей, заштрихованных на рисунке 23.

Учебное пособие по математике





Координаты и векторы

31. На рисунке 24 изображено колесо карусели, закрепленное в центре O планкой AB, причем AO = OB. По периметру колеса расположены кресла. Докажите, что сумма квадратов расстояний от них до концов планки есть величина постоянная.

Учебное пособие по математике

32. Проводятся соревнования по ориентированию на местности. Участникам раздали карты, на которых пункты сбора отмечены в вершинах четырехугольника, имеющих координаты A(-2, -5), B(-5, 3), C(3, 9), D(8, -3). Спортсмены разбиты на четыре команды, которые должны начать движение из точек E, F, G и H - середин соответствующих сторон AB, BC, CD и AD четырехугольника - в пункты, расположенные в серединах отрезков, соединяющих середины противоположных сторон четырехугольника. Изобразите план и определите координаты точек E, F, G, H, точек - пунктов встреч, а также расстояние, которые прошли команды до места встреч.

33. Определите расстояние между пунктом O и недоступной точкой M, если известны расстояния ( в км) OA, OB и OC, где A, B, C - пункты, расположенные в вершинах треугольника, центроидом которого является точка M. Точки имеют следующие координаты: O(0, 0), A(-7, 8), B(8, 15), C(-4, 1).

34. Участок ABC треугольной формы разделили на две равновеликие части, проведя межу AO (рис. 25). В свою очередь AO тоже разделили пополам вехойM и провели межу BM, которая пересекла AC в точке D. Докажите, что Учебное пособие по математике.

35. К одной точке приложены три силы P1 = 30H, P2 = 50H, P3 = 30H, располагающиеся в одной плоскости. Углы между соседними силами равны 120. Найдите величину и направление равнодействующей силы.

36. Какую силу нужно приложить к прямолинейному направлению движения под углом: а) 45; б) 60, чтобы на пути в 1 км она совершила работу, равную 200 Дж?

37. Под каким углом нужно приложить силу в 20 H к прямолинейному направлению движения, чтобы на пути в 50 м она совершила работу в 100 Дж?

38. Какой угол образуют единичные векторы Учебное пособие по математике и Учебное пособие по математике, если известно, что векторы Учебное пособие по математике + 2Учебное пособие по математике и 5Учебное пособие по математике - 4Учебное пособие по математикеперпендикулярны.

39. Какой геометрический смысл имеет формула: а) (Учебное пособие по математике + Учебное пособие по математике)(Учебное пособие по математике - Учебное пособие по математике) = Учебное пособие по математике2 + Учебное пособие по математике2; б) (Учебное пособие по математике + Учебное пособие по математике)2 + (Учебное пособие по математике - Учебное пособие по математике)2 = 2(Учебное пособие по математике2 + Учебное пособие по математике2)?

40. Под каким углом видит путешественник береговую полосу AB с корабля K, если KAB - равнобедренный треугольник, медианы которого, проведенные к боковым сторонам перпендикулярны (рис. 26).

Учебное пособие по математике




Тригонометрия


41. Тропинка длиной 1,8 км, поднимаясь в гору, образует с горизонтом угол 6. На какой высоте от подошвы горы находится база альпинистов, расположенная на вершине горы.

42. Объясните, как можно найти высоту дерева, используя прибор - высотомер, изображенный на рисунке 27.

43. Самолет был обнаружен вертикальным лучом прожектора, расположенным в 1,5 км от аэропорта. В то же время диспетчер в аэропорту увидел этот самолет под углом в 30. Найдите высоту, на которой находился самолет в этот момент и расстояние от самолета до аэропорта.

44. Самолет находится на высоте 7000 м и приближается к аэропорту. Для посадки летчик должен производить снижение под постоянным углом 6. На каком расстоянии от посадочной полосы он должен начать снижение?

45. Ширина дачного домика равна 6 м, ширина одного ската его двускатной крыши равна 5 м. Под каким углом к потолку поставлены стропила крыши?

46. На склоне холма стали рыть туннель длиной 0,5 км по направлению, составляющему с горизонтом угол 5. На каком расстоянии от поверхности холма будет находиться конец туннеля, если скат холма образует с горизонтом угол наклона 30?

47. Для определения высоты колонны поступили следующим образом: отошли от ее основания на 100 м, поставили угломерный прибор высотой 1,6 м и установили, что вершина колонны видна под углом 22. Найдите высоту колонны.

48. Из некоторой точки вершина горы видна под углом 30. При приближении к горе на 0,5 км вершина стала видна под углом 45. Найдите высоту горы.

49. Корабль движется на восток со скоростью 16 узлов (один узел равен одной морской миле в час, одна морская миля равна 1,852 км). В 12 часов азимут направления на маяк составил 60, а в 12 часов 30 минут составил 30. Определите расстояние, на котором находился корабль от маяка в 12 часов 30 минут.

50. Из городов A и C одновременно выезжают два поезда в направлениях AB и CD. Скорость первого поезда 80 км/ч, второго - 100 км/ч. Направления движения AB и CD пересекаются в точке M под углом 60, причем AM = 150 км, CM = 120 км. Определите, через какое время от начала движения поезда удалятся друг от друга на расстояние, равное расстоянию между городами A и C.


























ЭЛЕМЕНТЫ СТЕРЕОМЕТРИИ

1. В кубе ABCDA1B1C1D1 (A…D1) запишите три пары параллельных ребер.

Замечание. Два отрезка называются параллельными или скрещивающимися, если они лежат соответственно на параллельных или скрещивающихся прямых.

2. В тетраэдре ABCD запишите все пары скрещивающихся ребер.

3. Какие две прямые будут непараллельными: а) на плоскости; б) в пространстве?

4. Сколько плоскостей можно провести через две параллельные прямые?

5. Верно ли утверждение: «Если прямая пересекает одну из двух параллельных прямых, она пересекает и другую прямую: а) на плоскости; б) в пространстве»?

6. Исследуйте все возможные расположения трех попарно пересекающихся прямых: а) на плоскости; б) в пространстве.

7. Как расположены на рисунке 59 прямые a и b?

Учебное пособие по математике

8. Изобразите следующую геометрическую ситуацию: прямая a лежит в плоскости , а прямая b пересекает плоскость в точке B. Как могут располагаться относительно друг друга прямые a и b?

9. Изобразите следующую геометрическую ситуацию: сторона AD параллелограмма ABCD лежит в плоскости , плоскости ABC и не совпадают. Какое предположение можно сделать о взаимном расположении прямой BC и плоскости ? Как расположены относительно плоскости прямые BA и DC?

10. Условие предыдущей задачи 9, но ABCD - трапеция с основаниями AD и BC.

11. Сделайте предположение о параллельности ребра и грани в параллелепипеде A…D1. Запишите три соответствующие пары.

Замечание. Ребро многогранника называется параллельным его грани, если прямая, на которой лежит ребро, параллельна плоскости, в которой лежит эта грань.

12. Как могут располагаться относительно друг друга две плоскости?

13. Запишите параллельные грани правильного: а) гексаэдра A…D1; б) октаэдра SABCDS'. Сколько пар параллельных граней получилось?

Замечание. Грани многогранника называются параллельными, если они лежат в параллельных плоскостях.

14. Какое наименьшее число красок разных цветов нужно взять, чтобы правильно окрасить поверхность: а) тетраэдра; б) куба; в) октаэдра?

Замечание. Правильной называется окраска, при которой соседние грани многогранника окрашены в разные цвета.

15.Плоскость пересекает плоскости и по параллельным прямым. Обязательно ли плоскости и параллельны?

16. Как могут быть расположены три плоскости относительно друг друга, если две из них параллельны?

17. Найдите наибольшее число прямых, по которым могут попарно пересекаться: а) две; б) три; в) четыре плоскости.

18. Могут ли: а) три; б) четыре; в) n плоскостей пересекаться по одной прямой? Сделайте соответствующий рисунок.

19. Нарисуйте две различные развертки одного и того же правильного тетраэдра.

20. Нарисуйте три различные развертки одного и того же гексаэдра.

21. Нарисуйте три различные развертки одного и того же октаэдра.

22. Нарисуйте многоугольник, который образован четырьмя правильными равными треугольниками, но не является разверткой правильного тетраэдра.

23. Нарисуйте три многоугольника, каждый из которых состоит из шести равных квадратов, и которые не являются развертками куба.

24. Нарисуйте развертку правильной пирамиды, которая имеет: а) 5 вершин; б) 12 ребер.

25. Нарисуйте развертку правильной призмы, которая имеет: а) 6 вершин; б) 12 ребер.

26. Нарисуйте и назовите выпуклый многогранник, у которого: а) 7 вершин; б) 18 ребер; в) 5 граней; г) 8 граней.

27. Как могут располагаться относительно друг друга плоскость и многогранник? Изобразите соответствующие геометрические ситуации.

28. В каком случае говорят о сечении многогранника плоскостью?

29. Какой фигурой является сечение многогранника плоскостью?

30. Многоугольники с каким числом сторон могут получиться в сечениях: а) тетраэдра; б) треугольной призмы; в) четырехугольной пирамиды; г) куба?

31. В правильном единичном тетраэдре плоскость отсекает одну из вершин и проходит через середины ребер, выходящих из нее. Как называются два получившихся многогранника? Сделайте соответствующие рисунки. Опишите их грани и найдите периметры этих граней.

32. Куб пересечен плоскостью, которая проходит через: а) концы ребер; б) середины ребер, выходящих из одной вершины. Какой многоугольник получится в сечении? Сделайте рисунок и найдите периметр сечения, если ребро куба равно 2 см.

33. Как изменится число вершин (В), ребер (Р) и граней (Г) выпуклого многогранника, если к одной из его граней пристроить пирамиду?

34. Как изменится число вершин (В), ребер (Р) и граней (Г) выпуклого многогранника, если от него отсечь один из многогранных углов?

35. На рисунке 60 изображен многогранник, который называется усеченным тетраэдром. Все его ребра равны. Поясните, каким образом можно получить его из правильного тетраэдра? Сколько у него вершин (В), ребер (Р) и граней (Г), какого они вида?

Учебное пособие по математике

36. Каким образом из правильного: а) октаэдра; б) икосаэдра получить усеченный полуправильный многогранник (усеченные соответственно октаэдр и икосаэдр)? Из каких граней он состоит, сколько у него вершин (В), ребер (Р) и граней (Г)?

37. Из каких многоугольников состоит поверхность полуправильного многогранника: а) усеченного куба; б) усеченного додекаэдра? Найдите число его вершин (В), ребер (Р) и граней (Г).

38. Найдите сумму всех плоских углов (углов граней многогранника): а) тетраэдра; б) параллелепипеда; в) четырехугольной пирамиды; г) пятиугольной усеченной пирамиды; д) октаэдра; е) усеченного куба.

39. В кубе A…D1 определите вид четырехугольника KLMN, где K, L, M, N - середины соответственно ребер A1B1, C1D1, CD, AB, KLM= 90.

40. Найдите углы треугольников, вершинами которых являются вершины A1, D, C1 и A, C, D1 единичного куба A…D1. Будут ли эти треугольники равны? Найдите их периметры.

41. Определите вид треугольника AOC, где A, C - вершины куба A…D1 , O - середина ребра BB1. Сделайте соответствующий рисунок. Найдите углы треугольника, если ребро куба равно 1.

42. Определите вид треугольника ADH, где A, D - вершины правильного тетраэдра ABCD, H - середина ребра BC. Сделайте соответствующий рисунок. Найдите периметр и углы треугольника, если ребро тетраэдра равно a.

43. На рисунке 61 изображена правильная пирамида SABC, SA1:A1A=SB1:B1B=SC1:C1C=1:3. Запишите несколько пар подобных треугольников. Сколько всего пар подобных треугольников?

Учебное пособие по математике

44. В задаче 32 сравните полученные в случаях а) и б) в сечениях многоугольники, будут ли они подобны?

45. В четырехугольной пирамиде MABCD основание ABCD: а) параллелограмм; б) ромб. Сечение прошло через точки G, H, P, O - середины боковых ребер. Определите вид четырехугольника GHPO.

46. В прямой призме ABCA1B1C1, у которой AB=13 см, BC=14 см, AC=15 см и AA1=20 см, сечение проходит через точки K, L, M, N - середины ребер соответственно AB, AC, A1C1, A1B1. Сделайте рисунок. Найдите периметр полученного сечения и определите, подобны ли прямоугольники: а) KLMN иBCC1B1; б) AA1B1B и AA1C1C; в) BB1NK и CC1ML.

47. В основание ABC треугольной пирамиды SABC (рис. 62) вписана окружность. Будет ли SB секущей этой окружности?

48. Определите радиус окружности, вписанной в грань ABB1A1 единичного куба A…D1.

49. Дана правильная пирамида PABCD с вершиной в точке P, все ребра которой равны 3 см. Найдите радиусы окружностей, вписанных в треугольники PAC и PBD. Какой фигурой является пересечение данных окружностей?

50. Какие геометрические фигуры в пространстве аналогичны соответственно окружности и кругу на плоскости? Сформулируйте их определения.

Замечание. Окружностью называется геометрическая фигура, состоящая из всех точек плоскости, одинаково удаленных от данной точки. Эта точка называется центром окружности. Кругом называется геометрическая фигура, ограниченная окружностью, т.е. состоящая из всех точек плоскости, расстояния от которых до центра окружности не превосходит (меньше либо равно) ее радиуса.

51. Что называется радиусом и диаметром сферы?

52. Представьте себе, что плоскость пересекает: а) сферу; б) шар. Сделайте предположение о том, какой фигурой является сечение.

53. Какое сечение сферы плоскостью имеет наибольшую длину? Найдите ее, если радиус сферы равен R.

54. В сфере провели два сечения через ее центр. Какой фигурой является их пересечение?

55. Представьте себе, что прямоугольник вращается вокруг своей стороны. Нарисуйте фигуру, которая при этом получается. Как она называется?

56. Представьте себе, что прямоугольник вращается вокруг прямой, соединяющей середины его противоположных сторон. Нарисуйте фигуру, которая при этом получается. Как она называется?

57. Представьте себе, что прямоугольный треугольник вращается вокруг своего катета. Нарисуйте фигуру, которая при этом получается. Как она называется?

58. Представьте себе, что равнобедренный треугольник вращается вокруг своей высоты, опущенной на основание. Нарисуйте фигуру, которая при этом получается. Как она называется?

59. Представьте себе, что полуокружность (полукруг) вращается вокруг своего диаметра. Нарисуйте фигуру, которая при этом получается. Как она называется?

60. Представьте себе, что окружность (круг) вращается вокруг своего диаметра. Нарисуйте фигуру, которая при этом получается. Как она называется?


ЭЛЕМЕНТЫ СТЕРЕОМЕТРИИ

1. Изобразите следующую геометрическую ситуацию: прямая a лежит в плоскости , прямая b пересекает плоскость в точке B, которая не принадлежит прямой a. Сделайте соответствующие записи с помощью математической символики.

2. Изобразите следующую геометрическую ситуацию: плоскости и пересекаются по прямой c, прямая l пересекает плоскости и соответственно в точках A и B, которые не принадлежат прямой c. Сделайте соответствующие записи с помощью математической символики.

3. Изобразите плоскости , , , , которые имеют общую прямую h.

4. Изобразите плоскости , , , которые имеют общую точку H.

5. Пусть плоскости , пересекаются по прямой c. Плоскость пересекает плоскости и соответственно по прямым a и b, причем a || c, b || c. Сделайте предположение о взаимном расположении прямых a и b.

6. Три вершины: а) треугольника; б) параллелограмма; в) пятиугольника принадлежат плоскости. Будет ли вся фигура лежать в этой плоскости?

7. Три точки: а) окружности; б) квадрата; в) трапеции принадлежат плоскости. Будет ли вся фигура лежать в этой плоскости?

8. Изобразите ромб ABCD, у которого: а) вершины A и D принадлежат плоскости и плоскость ромба не совпадает с данной плоскостью; б) вершины A и Cпринадлежат плоскости и плоскость ромба не совпадает с данной плоскостью; в) вершины B, C и O - точка пересечения диагоналей принадлежат плоскости .

9. Какой многоугольник лежит в основании: а) призмы; б) пирамиды, если у нее 60 ребер?

10. Найдите сумму всех плоских углов: а) куба; б) октаэдра; в) пятиугольной пирамиды; г) десятиугольной призмы.

11. Сколько красок потребуется для правильной раскраски граней: а) призмы с четным числом боковых ребер; б) пирамиды с четным числом боковых ребер; в) призмы с нечетным числом боковых ребер; г) пирамиды с нечетным числом боковых ребер; д) бипирамиды с четным числом ребер в общем основании; е) бипирамиды с нечетным числом ребер в общем основании.

12. В правильном единичном тетраэдре ABCD найдите углы: а) ADB; б) ABC; в) CMD, где M - середина ребра AC; г) DHC, где H - середина ребра AB; д)AKB, где K - середина ребра CD.

13. Найдите углы между пересекающимися диагоналями граней прямоугольного параллелепипеда с измерениями 7 см, 7Учебное пособие по математикесм, 7 см.

14. Изобразите многогранники, имеющие: а) трехгранные; б) четырехгранные; в) пятигранные углы.

15. Сколько плоскостей можно провести через различные пары из: а) двух; б) трех; в) четырех; г) n параллельных прямых, никакие три из которых не лежат в одной плоскости?

16. На рисунке 28 изображена правильная шестиугольная призма. Запишите все ребра: а) параллельные ребру AB; б) параллельные ребру CC1; в) скрещивающиеся с ребром EF; г) скрещивающиеся с ребром BB1.

Учебное пособие по математике

17. Изобразите четырехугольную пирамиду: а) выпуклую; б) невыпуклую.

18. Изобразите пятиугольную призму: а) выпуклую; б) невыпуклую.

19. Изобразите невыпуклый многогранник, все грани которого являются выпуклыми многоугольниками.

20. Будет ли треугольная бипирамида, составленная из правильных тетраэдров правильным многогранником. Почему?

21. Изобразите: а) куб; б) октаэдр; в) тетраэдр и двойственный к нему многогранник.

22. Найдите сумму плоских углов каждого правильного многогранника.

23. Найдите число двугранных углов каждого правильного многогранника.

24. Определите число вершин (В), ребер (Р) и граней (Г) усеченного: а) тетраэдра; б) гексаэдра; в) октаэдра; г) додекаэдра; д) икосаэдра.

25. Нарисуйте куб и полученный из него кубооктаэдр. Найдите ребро кубооктаэдра, если ребро куба равно 4 см. Определите число его вершин, ребер и граней.

26. Изобразите многогранник, который не является ни правильным, ни полуправильным, но все грани которого - правильные многоугольники.

27. Изобразите вписанный в куб: а) тетраэдр; б) многогранник Кеплера - "Stella Octangula", найдите его ребро, если ребро куба равно a. Какой многогранник является пересечением тетраэдров, образующих многогранник Кеплера?

28. Нарисуйте куб и полученный из него усеченный куб Найдите ребро усеченного куба, если ребро куба равно b.

29. Поверхность какого многогранника напоминает поверхность футбольного мяча? Сколько у этого многогранника граней и какого они вида?

30. Изобразите многоугольник, который состоит из: а) четырех равных правильных треугольников; б) шести равных квадратов; в) восьми правильных треугольников и является разверткой соответственно правильного: а) тетраэдра; б) гексаэдра; в) октаэдра.

31. Изобразите многоугольник, который состоит из: а) четырех равных правильных треугольников; б) шести равных квадратов; в) восьми правильных треугольников и не является разверткой соответственно правильного: а) тетраэдра; б) гексаэдра; в) октаэдра.

32. Изобразите развертку правильной: а) треугольной призмы; б) четырехугольной пирамиды.

33. Изобразите развертку: а) правильной четырехугольной призмы; б) прямоугольного параллелепипеда; в) наклонного параллелепипеда.

34. Изготовьте модели многогранников по разверткам, изображенным на рисунке 29. Как они называются?

Учебное пособие по математике

35. Каким образом из правильной n-угольной призмы получить соответствующий полуправильный многогранник - n-угольную антипризму, если: а) n = 4; б) n = 5; в) n = 6; г) n = m?

36. Какое число цветов нужно взять, чтобы правильно окрасить поверхность многогранников из задачи 34?

37. Изобразите развертку: а) цилиндра; б) конуса.

38. Найдите площадь поверхности: а) куба; б) правильного тетраэдра; в) октаэдра, г) икосаэдра, если их ребра равны 5 дм.

39. Найдите площадь: а) боковой; б) полной поверхности прямой треугольной призмы, если ее боковое ребро равно 15 см, а стороны основания равны 10 см, 12 см и 10 см.

40. Найдите площадь: а) боковой; б) полной поверхности правильной треугольной пирамиды, если ее боковое ребро равно 13 см, а апофема (высота боковой грани, проведенная к основанию) - 5 см.

41. Площадь боковой поверхности правильной треугольной призмы равна 480 см2. Высота призмы относится к стороне основания как 8 : 5. Найдите высоту призмы и сторону основания.

42. Сколько прямоугольных листов жести размером 2 м x 1 м понадобится для покрытия крыши башни, имеющей форму пирамиды с квадратным основанием, сторона которого равна 2,5 м, а длина ската крыши равна 3 м? Учесть, что на швы и обрезки пойдет 0,5 листа.

43. Сколько потребуется краски, чтобы покрасить печь цилиндрической формы, диаметр основания которой равен 15 дм, а высота 30 дм, если на один квадратный метр расходуется 200 г?

44. Высота конуса равна 24 см, радиус основания равен 7 см. Найдите площадь: а) боковой; б) полной поверхности конуса.

45. Сколько досок длиной 3,5 м, шириной 20 см и толщиной 20 мм выйдет из четырехугольной балки длиной 105 дм, имеющей в сечении прямоугольник размером 30 см x 40 см?

46. Рассчитайте, какова должна быть площадь кабинета высотой 3,5 м для класса в 28 человек, если на каждого ученика нужно 7,5 м3 воздуха.

47. Найдите объем и площадь поверхности правильной шестиугольной призмы, если сторона основания относится к боковому ребру как 2 : 5 и радиус вписанной в основание окружности равен Учебное пособие по математикесм.

48. Из медного листа толщиной 4 мм вырезали квадрат, правильные треугольник и шестиугольник, каждый многоугольник со стороной 10 см. Какие многогранники получились и как относятся их объемы?

49. Запишите формулу объема цилиндра: а) через его высоту (h) и длину окружности основания c; б) диаметр D основания которого равен его образующей.

50. Сколько чугунных труб длиной 4 м с наружным диаметром 33,5 мм и толщиной 3,25 мм можно погрузить на четырехтонный грузовик (удельный вес чугуна 7,3 г/см3)?













Учебное пособие по математике

РАЗДЕЛ 2. Математическая логика

Учебное пособие по математике

«Алгебра логики»



учащиеся должны научиться применять законы алгебры логики и технологию комплексного использования ЭВМ при решении логических задач.

Логика - это наука о формах и законах человеческого мышления. В зависимости от набора правил вывода умозаключений различают несколько вариантов логики: формальная, математическая, вероятностная, диалектическая.

Формальная логика связана с анализом наших обычных содержательных рассуждений, выражаемых разговорным языком.

Основоположником формальной логики является Аристотель, который впервые отделил логические формы мышления от его содержания.

Математическая логика, являясь частью формальной логики, изучает только суждения и рассуждения, для которых можно однозначно решить: истинны они или ложны.

Введение
Процесс информатизации общества, связанный с бурным развитием информационных и коммуникационных технологий, их аппаратной базы и программного обеспечения, объективно ставят перед современным специалистом любого профиля задачу рационализации интеллектуальной деятельности на основе внедрения достижений информатики и математики.

В данный период развития общества производство информации становится основным видом деятельности. Проблемы «искусственного интеллекта», которые актуальны в настоящее время, основаны полностью на математической логике. Изучение раздела математической логики является определяющим на этапе внедрения информационных технологий в профильное обучение в образовательных учреждениях, что позволяет качественно улучшить процесс образования.

Развитие предметной области ставит перед системой образования задачи:

развитие культуры мышления или познания;
развитие творческого потенциала;
умение пользоваться современными вычислительными средствами; разработки критериев оценки качества подготовки выпускников учебных заведений всех уровней к жизни и труду в современном информационном обществе.


Поэтому настоящая программа актуальна.

Новизна программы
Логика является основным инструментом при решении, как математических задач, так и задач по программированию, а программирование является «стержнем информатики». Алгебра логики лежит в основе теории искусственного интеллекта. Умение логически мыслить позволяет работать творчески в любой области знаний. Результаты работы специалистов, работающих на «стыке» математики и информатики, достижения в вычислительной технике, огромный опыт формализации и решения сложнейших проблем в самом программировании, связанный с созданием больших программных комплексов с использованием современных принципов программирования, основанных на глубоком знании математики, позволяет строить модели знаний, являющиеся основой компьютерных систем искусственного интеллекта. Учащиеся получают навыки решения логических задач различными способами, используя ЭВМ, логические высказывания и выражения.

Таким образом данная система решает несколько задач:


  • развитие логического мышления;


  • знакомство с основами искусственного интеллекта;


  • использование компьютерных технологий для решения логических задач;


  • применение и сравнение различных способов решения логических задач.


Методологическое обоснование программы
Данный материал знакомит не только с булевой алгеброй, но и с практическим применением полученных знаний при решении логических задач разными методами. Изучение начинается с введения понятий алгебры логики. На этом этапе особое внимание уделяется основам современной логики. Рассматриваются основные формы мышления, логические операции и действия над ними.
Следующий блок посвящён изучению способов решения логических задач.
Еще один блок посвящён изучению связи между алгеброй логики и двоичным кодированием.
Цель: развитие логического и творческого мышления учащихся: от алгоритмического к структурному, а затем к эвристическому мышлению.

Учебное пособие по математике

Метод логических квадратов

Задача 1: В кафе встретились три друга: скульптор Белов, скрипач Чернов и художник Рыжов. «Замечательно, что один из нас блондин, другой - брюнет, а третий - рыжий, и при этом ни у одного из нас цвет не соответствует фамилии», - заметил черноволосый. «Ты совершенно прав», - сказал Белов. Определите цвет волос художника.

Задача 2: Три подруги были на выпускном балу в белом, красном и голубом платье. Их туфли были тех же трёх цветов. Только у Тамары цвета платья и туфель совпадали. Валя была в белых туфлях. Ни платье, ни туфли Лиды не были красными. Определите цвета платьев и туфель у подруг.

Задача 3: Кондратьев, Давыдов и Федоров живут на одной улице. Один из них - столяр, другой - маляр, третий - водопроводчик. Недавно маляр хотел попросить своего знакомого столяра сделать кое-что для своей квартиры, но ему сказали, что столяр работает в доме водопроводчика. Известно также, что Федоров никогда не слышал о Давыдове. Кто чем занимается?

Задача 4: Три товарища - Владимир, Игорь и Сергей - окончили один и тот же педагогический институт и преподают математику, физику и литературу в школах Тулы, Рязани и Ярославля. Владимир работает не в Рязани, Игорь - не в Туле. Рязанец преподает не физику, Игорь - не математику, туляк преподает литературу. Какой предмет и в каком городе преподает каждый из них?

Задача 5: Познакомимся с тремя людьми: Алешиным, Беляевым и Белкиным. Один из них - архитектор, другой - бухгалтер, третий - археолог. Один живет в Белгороде, другой - в Брянске, третий в Астрахани. Требуется узнать, кто где живет и у кого какая профессия.

Белкин бывает в Белгороде лишь наездами и то весьма редко, хотя все его родственники постоянно живут в этом городе.

У двух из этих людей названия профессий и городов, в которых они живут, начинаются с той же буквы, что и их имена.

Жена архитектора доводится Белкину младшей сестрой.

Задача 6: На улице, став в кружок, разговаривают четыре девочки: Аня, Валя, Галя и Нина. Девочка в зеленом платье (не Аня и не Валя) стоит между девочкой в голубом платье и Ниной. Девочка в белом платье стоит между девочкой в розовом платье и Валей. Какое платье на каждой из девочек?

Задача 7: За круглым столом сидели четыре студента. Филолог сидел против Козина, рядом с историком. Математик сидел рядом с Волковым. Соседи Шатрова - Егоркин и физик. Какая профессия у Козина?

Задача 8: Петр, Геннадий, Алексей и Владимир занимаются в детской спортивной школе в разных секциях: гимнастики, легкой атлетики, волейбола и баскетбола. Петр, Алексей и волейболист учатся в одном классе. Петр и Геннадий на тренировки ходят пешком вместе, а гимнаст ездит на автобусе. Легкоатлет не знаком ни с волейболистом, ни с баскетболистом. Кто в какой секции занимается?

Задача 9: Среди офицеров А, Б, В и Г - майор, капитан и два лейтенанта. А и один из лейтенантов - танкисты, Б и капитан - артиллеристы, А младше по званию, чем В. Определите род войск и звание каждого.

Задача 10: Корнеев, Докшин, Мареев и Скобелев - жители нашего города. Их профессии - пекарь, врач инженер и милиционер.

Корнеев и Докшин - соседи и всегда на работу ездят вместе

Докшин старше Мареева

Корнеев регулярно обыгрывает Скобелева в пинг-понг

Пекарь на работу всегда ходит пешком

Милиционер не живет рядом с врачом

Инженер и милиционер встречались единственный раз, когда милиционер оштрафовал инженера за нарушение правил уличного движения.

Милиционер старше врача и инженера

Определите, кто чем занимается.

Задача 11: Борисов, Кириллов, Данин и Савин - инженеры. Один из них - автомеханик, другой - химик, третий - строитель, четвертый - радиотехник.

Борисов, который обыгрывает в шахматы Данина, но проигрывает Савину, бегает на лыжах лучше того инженера, который моложе его, и ходит в театр вдвое чаще, чем тот инженер, который старше Корнилова.

Химик, который посещает театр вдвое чаще, чем автомеханик, не является ни самым молодым, ни самым пожилым из этой четверки.

Строитель, который бегает на лыжах хуже, чем радиотехник, как правило, проигрывает в шахматных сражениях автомеханику.

Самый пожилой из инженеров лучше всех играет в шахматы и чаще всех бывает в театре, а самый молодой лучше всех ходит на лыжах.

Назовите профессии каждого из этой четверки инженеров, если известно, что ни в спорте, ни в приверженности к театру среди них нет двух одинаковых,

Задача 12: Дина, Соня, Коля, Рома и Миша учатся в институте. Их фамилии - Бойченко, Карпенко, Лысенко, Савченко и Шевченко.

Мать Ромы умерла.

Родители Дины никогда не встречались с родителями Коли.

Студенты Шевченко и Бойченко играют в одной баскетбольной команде.

Услышав, что родители Карпенко собираются поехать за город, мать Шевченко пришла к матери Карпенко и попросила, чтобы та отпустила своего сына к ним на вечер, но оказалось, что отец Коли уже договорился с родителями Карпенко и пригласил их сына к Коле.

Отец и мать Лысенко - хорошие друзья родителей Бойченко. Все четверо очень довольны, что их дети собираются пожениться.

Установите имя и фамилию каждого из молодых людей и девушек.

Задача 13: В семье Семеновых 5 человек: муж, жена, их сын, сестра мужа и отец жены. Все они работают. Один - инженер, другой - юрист, третий - слесарь, четвертый - экономист, пятый - учитель. Вот что еще известно о них. Юрист и учитель не кровные родственники. Слесарь - хороший спортсмен. Он пошел по стопам экономиста и играет в футбол за сборную завода. Инженер старше жены своего брата, но моложе, чем учитель. Экономист старше, чем слесарь. Назовите профессии каждого члена семьи Семеновых.

Задача 14: Поездная бригада состоит из кондуктора, проводника, машиниста и помощника машиниста. Их зовут Андрей, Петр, Дмитрий и Трофим.

Дмитрий старше Андрея.

У кондуктора нет родственников в бригаде.

Машинист и помощник машиниста - братья. Других братьев у них нет.

Дмитрий - племянник Петра.

Помощник машиниста - не дядя проводника, а проводник - не дядя машиниста.

Кто в качестве кого работает? Какие родственные отношения существуют между членами бригады?



Логическая арифметика

Задача 1: Заполните пропуски так, чтобы полученное предложение было а) истинным; б) ложным.

1) «Число 24 делится на 3 и на …»

2) «число 11 делится на 3 и на …»

Задача 2: Для каких натуральных n предложение «n - число чётное и n + 2k + 1 также чётное число» является истинным?

Задача 3: Для каких натуральных n предложение «n - число чётное или нечётное» является истинным?

Задача 4: Сформулируйте с помощью союза «или» утверждение «по крайней мере одно из чисел n, n + 1, n + 2 является чётным». Истинно ли это предложение?

Задача 5: Сформулируйте с помощью союза «или» и других союзов утверждение «ровно одно из чисел n, n + 1, n + 2 делится на 4». Истинно ли это предложение?

Задача 6: Сформулировать отрицание следующих предложений:

1) в праздники я схожу в кино по крайней мере один раз;

2) я никогда не был в Одессе;

3) за время каникул он был в музее, сходил в кино, а также два раза ходил в парк;

4) каждый человек на Земле хочет мира.

Задача 7: Какое из предложений: «a < 2», «a не больше 2", является отрицанием предложения «2 < a»? Чем они отличаются?

Задача 8: Является ли предложение «он мой враг» отрицанием предложения «он мой друг»?

Задача 9: Учитель сказал: «Кто закончит учебный год без троек, тот поедет в Москву на экскурсию». У Васи было две тройки, которые он не смог исправить. Значит ли это, что он не поедет в Москву?

Задача 10: Перед дождем Петин кот всегда чихает. Сегодня он чихнул. «Значит, будет дождь,» - подумал Петя. Прав ли он?

Задача 11: Если ученик много занимается, то он успешно сдает экзамены. Ученик провалился на экзаменах. Значит ли это, что он мало занимался?

Задача 12: Верны ли следующие предложения:

1) если 2 × 2 = 4, то 2 = 2;

2) если 2 × 2 = 4, то 2 = 0;

3) если 2 × 2 = 5, то 2 = 2;

4) если 2 × 2 = 5, то 2 = 0?

Задача 13: Равносильны ли следующие утверждения:

1) «x(x - 1) = 0»; «x = 0 и x = 1".

2) «x(x - 1) = 0»; «x = 0 или x = 1".

3) «x > 3»; «x > 5».

4) «сегодня пятница»; «завтра суббота»; «послезавтра воскресенье».

5) «Кто не рискует, тот не пьёт шампанского»; «Кто пьёт шампанское, тот рискует».

Задача 14: Если мама поедет в командировку, то Коля поедет в лагерь. Мама поедет в командировку или в дом отдыха. Если мама поедет в дом отдыха, то Лена поедет к бабушке. Но Лена не поедет к бабушке. Следовательно, Коля поедет в лагерь. Верно ли это рассуждение?

Задача 15: Сформулируйте отрицания следующих предложений.

1) все политики - мошенники;

2) все политики - люди богатые и нечестные;

3) нет ни одного бедного и честного политика;

4) среди политиков нет бедных людей, а также и честных тоже;

5) есть честные политики;

6) Явлинский - честный политик;

7) Жириновский - политик, и, следовательно, обманывает простых людей;

8) если человек жулик, то его фамилия - не Жириновский;

9) неверно, что все политики продажны или богаты;

10) не все политики продажны, да и богаты тоже не все;

11) неверно, что если человек политик, то он жулик;

12) бывают честные политики.

Задача 16: Найдите среди высказываний из предыдущей задачи равносильные, противоречащие друг другу, пары высказываний, в которых одно следует из другого, и пары предложений, в которых одно является отрицанием другого.









Задачи для самостоятельного решения на логику

Задача 1: В пяти корзинах А, Б, В, Г и Д лежат яблоки пяти разных сортов. В каждой из корзин А и Б находятся яблоки 3 и 4 сорта, в корзине В - 2 и 3, в корзине Г - 4 и 5, в корзине Д - 1 и 5. Занумеруйте корзины так, чтобы в первой корзине имелись яблоки 1 сорта (как минимум одно), во второй корзине - яблоки второго сорта и т.д.

Задача 2: Разбирается дело Брауна, Джонса и Смита. Один из них совершил преступление. В ходе следствия каждый из них сделал по два заявления. Браун: »Я не делал этого. Джонс не делал этого». Смит: «Я не делал этого. Это сделал Браун.» Джонс: «Браун не делал этого. Это сделал Смит.» Потом оказалось, что один из них дважды сказал правду, другой - дважды солгал, третий - раз сказал правду, раз солгал. Кто совершил преступление?

Задача 3: Один из пяти братьев испек маме пирог. Андрей сказал: «Это Витя или Толя». Витя сказал: «Это сделал не я и не Юра». Толя сказал: «Вы оба шутите». Дима сказал: «Нет, один из них сказал правду, а другой - нет». Юра сказал: «Нет, Дима, ты не прав.» Мама знает, что трое из ее сыновей всегда говорят правду. Кто испек пирог?

Задача 4: Из утверждений «число a делится на 2», «число a делится на 4», «число a делится на 12», «число a делится на 24» три верных, а одно неверное. Какое именно?

Задача 5: Команды А, Б, В, Г и Д участвовали в эстафете. До соревнований пять болельщиков высказали следующие прогнозы.

1) команда Д займет 1 место, команда В - 2;

2) команда А займет 2 место, Г - 4;

3) В - 3 место, Д - 5;

4) В - 1 место, Г - 4 место;

5) А - 2 место, В - 3.

В каждом прогнозе одна часть подтвердилась, а другая - нет. Какое место заняла каждая из команд?

Учебное пособие по математике

РАЗДЕЛ 3. Решение нестандартных

текстовых задач.

Учебное пособие по математике
Исключение невозможных значений, подбор ответа, рекомендации по решению нестандартных задач, задачи на «числа»



Решение нестандартных задач позволяет сформировать у обучающихся устойчивые знания по предмету, развить мышление и интуицию, расширить кругозор. Особенностью курса является привитие навыков самостоятельности: рассуждениях, в поисках способов решения задач, при выполнении заданий.

К нестандартным задачам относятся задачи, решение которых основано на применении специальных замен переменных; задачи, для решения которых необходимо, прежде всего, умение проводить довольно разветвлённые логические построения; задачи, для решения которых применяются методы анализа функций и их графиков, с использованием понятий математического анализа, начиная с простейших, таких как область определения и область значений функции, её возрастание и убывание, чётность и периодичность, касательная к графику функции, а также задачи на максимумы и минимумы или наибольшее или наименьшее значения различных функций.

Наиболее трудной и важной частью решения нестандартных задач является исследование физических и геометрических закономерностей в зависимости от параметров. Именно задачи с параметрами дают наиболее полное представление о знании учащимися основных разделов школьной математики, уровне математического и логического мышления. Первоначальных навыков исследовательской деятельности.

Цель

Углубление знаний учащихся о различных методах решения уравнений и базовых математических понятий, используемых при обосновании того или иного метода решения; формирование у школьников компетенций, направленных на выработку навыков самостоятельной и групповой исследовательской деятельности.

Задачи

классификация способов решения нестандартных уравнений, углубление теоретических основ школьной математики для решения каждого вида уравнений.

интеллектуальное развитие учащихся, формирование качеств мышления, характерных для математической деятельности и необходимых человеку для полноценной жизни в обществе. Развитие мыслительных способностей учащихся: умение анализировать, сопоставлять, сравнивать, систематизировать и обобщать.

воспитание личности в процессе освоения математики и математической деятельности, развитие у учащихся самостоятельности и способности к самореализации.

Методические рекомендации

Основой может служить технология деятельностного метода, которая обеспечивает системное включение ребенка в процесс самостоятельного построения им нового знания и позволяет учителю проводить разноуровневое обучение. Занятия должны носить проблемный характер. Ученики самостоятельно, в микрогруппах, в сотрудничестве с учителем выполняют задания, предполагающие исследовательскую деятельность, на занятиях организуется обсуждение результатов этой работы.

Требования к уровню освоения

В результате изучения учащиеся должны овладеть следующими знаниями, умениями и способами деятельности:

*иметь представление о математике как форме описания и методе познания действительности;

*уметь анализировать, сопоставлять, сравнивать, систематизировать и обобщать;

*уметь самостоятельно работать с методической литературой;

знать основные приемы решения нестандартных уравнений, понимать теоретические основы способов решения уравнений;

*уметь решать нестандартные уравнения различными методами;

*уметь представлять результат своей деятельности, участвовать в дискуссиях;

*уметь проводить самоанализ деятельности и самооценку ее результата.



Учебное пособие по математике

О решении текстовых задач по математике



Задачи являются материалом для ознакомления учащихся с новыми понятиями, для развития логического мышления, формирования межпредметных связей. Задачи позволяют применять знания, полученные при изучении математики, при решении вопросов, которые возникают в жизни человека. Этапы решения задач являются формами развития мыслительной деятельности.

Для решения текстовых задач применяются три основных метода: арифметический, алгебраический и комбинированный. Рассмотрим каждый из этих методов.



I. Арифметический метод.

Первым этапом решения задач арифметическим методом является разбор условия задачи и составление плана её решения. Этот этап решения задачи сопровождается максимальной мыслительной деятельностью.

Вторым этапом является решение задачи по составленному плану. Этот этап решения проводится учащимися без особых затруднений и в большинстве случаев носит тренировочный характер.

Третьим важным этапом решения задачи является проверка решения задачи. Она проводится по условию задачи. Пренебрежение проверкой при решении задачи, замена её проверкой ответов снижает роль решения задачи в процессе развития логического мышления учащихся.

При решении текстовых задач арифметическим методом у учащихся вырабатываются определённые умения и навыки, которые в процессе дальнейшего обучения должны совершенствоваться и закрепляться.

При арифметическом методе решения задач формируются 56 основных умений и навыков. Из них 38 умений и навыков приобретаются при решении задач как арифметическим, так и алгебраическим методами.

К ним относятся следующие умения и навыки:

  1. Краткая запись условия задачи.

  2. Изображение условия задачи с помощью рисунка.

  3. Логические приёмы мышления: наблюдение и сравнение, анализ и синтез, абстрагирование и конкретизация, обобщение и ограничение, умозаключения индуктивного и дедуктивного характера и умозаключения по аналогии.

  4. Выполнение арифметических действий над величинами (числами).

  5. Изменение (увеличение или уменьшение) величины (числа) в несколько раз.

  6. Нахождение разностного сравнения величин (чисел).

  7. Нахождение кратного сравнения величин (чисел).

  8. Использование свойств изменения результатов действий в зависимости от изменения компонентов.

  9. Изменение (увеличение или уменьшение) величины (числа) на несколько единиц величины (числа).

  10. Нахождение дроби от величины (числа).

  11. Нахождение величины (числа) по данной её (его) дроби.

  12. Нахождение процентов данной величины (данного числа).

  13. Нахождение величины (числа) по её (его) проценту.

  14. Нахождение процентного отношения двух величин (чисел).

  15. Составление пропорций.

  16. Понятие прямой и обратной пропорциональной зависимости величин (чисел).

  17. Понятие производительности труда.

  18. Определение производительности труда при совместной работе.

  19. Определение части работы, выполненной в течение некоторого промежутка времени.

  20. Определение скорости движения.

  21. Определение пути, пройденного телом.

  22. Определение времени движения тела.

  23. Понятие о собственной скорости (скорости в стоячей воде) движения тела по воде.

  24. Нахождение пути, пройденного двумя телами при встречном движении.

  25. Нахождение скорости движения тела по течению и против течения реки.

  26. Нахождение времени прохождения телом единицы пути при заданной скорости движения.

  27. Нахождение скорости сближения тел, движущихся в одном направлении, и скорости удаления.

  28. Нахождение скорости сближения или скорости удаления тел, движущихся в противоположных направлениях или при встречном движении.

  29. Нахождение части пути, пройденного телом за определённое время, когда известно время прохождения всего пути.

  30. Нахождение количества вещества, содержащегося в растворе, смеси, сплаве.

  31. Нахождение концентрации, процентного содержания.

  32. Нахождение стоимости товара, акции.

  33. Нахождение цены товара, акции.

  34. Нахождение прибыли.

  35. Нахождение количества вредных веществ в воде, воздухе.

  36. Нахождение себестоимости продукции.

  37. Расчёт начислений банка на вклады.

  38. Проверка решения задачи по условию.

Умения и навыки, которые формируются в процессе решения задач только арифметическим методом, можно разбить на две группы. К первой группе относятся умения и навыки, которые необходимы для дальнейшего изучения математики.

К первой группе относятся следующие умения и навыки:

  1. Перевод календарного времени в арифметическое число.

  2. Перевод арифметического числа в календарное время.

  3. Нахождение времени предыдущего события.

  4. Нахождение времени последующего события.

  5. Нахождение промежутка времени между двумя событиями.

Все умения и навыки этой группы формируются в процессе решения задач на вычисление времени, т.е. тех задач, которые нет смысла решать алгебраически.

Вторая группа - это те умения и навыки, без знания которых можно решить все текстовые задачи алгебраическим методом, и в дальнейшем их незнание не будет пробелом в математическом образовании учащихся.

Ко второй группе относятся следующие умения и навыки:

  1. Введение понятия "часть".

  2. Выполнение действий сложения и вычитания частей.

  3. Выполнение умножения и деления части на число.

  4. Приём уравнивания большего числа с меньшим и меньшего с большим.

  5. Приём уравнивания прибавлением к меньшему числу и вычитанием из большего числа их полуразности.

  6. Определение числа частей, составляющих данное число.

  7. Введение понятий условной единицы.

  8. Нахождение дроби условной единицы и её частей.

  9. Сравнение частей величин.

  10. Сложение и вычитание частей единицы.

  11. Метод исключения неизвестного посредством замены одной величины другой.

  12. Решение задач методом предположения.

  13. Составление плана решения задачи.

Эти умения и навыки, несомненно, представляют интерес. Но почти все из них можно отнести к числу умений и навыков, формирующихся у учащихся при решении нестандартных задач. Решение таких задач следует проводить систематически наряду с решением стандартных текстовых задач.



II. Алгебраический метод.

Под алгебраическим методом решения задач понимается такой метод решения, когда неизвестные величины находятся в результате решения уравнения или системы уравнений, решения неравенства или системы неравенств, составленных по условию задачи. Иногда алгебраическое решение задачи бывает очень сложным.

При решении задач алгебраическим методом основная мыслительная деятельность сосредотачивается на первом этапе решения задачи: на разборе условия задачи и составлении уравнений или неравенств по условию задачи.

Вторым этапом является решение составленного уравнения или системы уравнений, неравенства или системы неравенств.

Третьим важным этапом решения задач является проверка решения задачи, которая проводится по условию задачи.

При алгебраическом методе решения формируются 55 основных умений и навыков.

Отличными от тех, которые формируются при арифметическом их решении, являются следующие:

  1. Введение неизвестного.

  2. Введение двух неизвестных.

  3. Введение трёх и более неизвестных.

  4. Выполнение действий сложения и вычитания неизвестных.

  5. Выполнение действий умножения и деления неизвестных.

  6. Запись зависимости между величинами с помощью букв и чисел.

  7. Решение линейных уравнений.

  8. Решение линейных неравенств.

  9. Решение квадратных уравнений и неравенств.

  10. Решение дробно-рациональных уравнений и неравенств.

  11. Решение систем уравнений и систем неравенств.

  12. Составление одного уравнения (неравенства) с двумя неизвестными.

  13. Решение уравнения (неравенства) с двумя неизвестными.

  14. Выбор значений неизвестных по условию задачи.

  15. Составление уравнений с параметром по условию текстовой задачи.

  16. Решение уравнений с параметром.

  17. Исследовательская работа.

III. Комбинированный метод.

Этот метод получается в результате включения в алгебраический метод решения задач решение, в котором часть неизвестных величин определяется с помощью решения уравнения или системы уравнений, неравенств или систем неравенств, а другая часть - арифметическим методом. В этом случае решение текстовых задач значительно упрощается.

При решении текстовых задач учащимся могут помочь несколько простых и общих советов, а также приведённые ниже примеры решения задач.

Совет 1. Не просто прочитайте, а тщательно изучите условие задачи. Попытайтесь полученную информацию представить в другом виде - это может быть рисунок, таблица или просто краткая запись условия задачи.

Совет 2. Выбор неизвестных.

В задачах "на движение" - это обычно скорость, время, путь. В задачах "на работу" - производительность и т.д.

Не надо бояться большого количества неизвестных или уравнений. Главное, чтобы они соответствовали условию задачи и можно было составить соответствующую "математическую модель" (уравнение, неравенство, система уравнений или неравенств).

Совет 3. Составление и решение "математической модели".

При составлении "математической модели" (уравнения, неравенства, системы уравнений или неравенств) ещё раз внимательно прочитайте условие задачи. Проследите за тем, что соответствует каждой фразе текста задачи в полученной математической записи и чему в тексте задачи соответствует каждый "знак" полученной записи (сами неизвестные, действия над ними, полученные уравнения, неравенства или их системы).

Очень важно не только составить уравнение, неравенство, систему уравнений или неравенств, но и решить составленное.

Если решение задачи не получается, то нужно ещё раз прочитать и проанализировать задачу (заданный текст и полученную запись).

Иногда по условию задачи достаточно отыскать не сами неизвестные, а их комбинации. Например, не xи y, а x+y, x/y, 1/x и т.п.

Если кажется, что получилось правильное, но очень сложное выражение, то попробуйте ввести другие неизвестные, может быть, изменив их количество, чтобы получилась более простая модель.

Иногда неизвестные в задачах выражаются только целыми числами, тогда при решении задач нужно использовать свойства целых чисел.

Совет 4. Решение сложной текстовой задачи - процесс творческий. Иной раз требуется вернуться к самому началу задачи, учитывая и анализируя уже полученные результаты.

При решении задач короткую запись задачи можно сделать с помощью рисунка или таблицы.

Можно выделить семь вопросов, которые дают верное направление решению задач разных типов.

Вопросы к задаче с комментариями к ним:

  1. О каком процессе идёт речь? Какими величинами характеризуется этот процесс? (Количество величин соответствует числу столбцов таблицы).

  2. Сколько процессов в задаче? (Количество процессов соответствует числу строк в таблице).

  3. Какие величины известны? Что надо найти? (Таблица заполняется данными задачи; ставится знак вопроса).

  4. Как связаны величины в задаче? (Вписать основные формулы, выяснить связи и соотношения величин в таблице).

  5. Какую величину (величины) удобно выбрать в качестве неизвестной или неизвестных? (Клетки в таблице заполняются в соответствии с выбранными неизвестными).

  6. Какие условия используются для составления "модели"? (Выписать полученную "модель")

  7. Легко ли решить полученное? (Если решить сложно, ввести новые переменные, использовать другие соотношения).

Учебное пособие по математике

Пример решения задачи.

Задача. Расстояние между двумя городами скорый поезд проходит на 4 часа быстрее товарного и на 1 час быстрее пассажирского. Найти скорости товарного и скорого поездов, если известно, что скорость товарного поезда составляет 5/8 от скорости пассажирского и на 50 км/ч меньше скорости скорого.

Решение (черновик).

Отвечаем на вопросы, поэтапно составляя таблицу.

1. Речь идёт о процессе движения, которое характеризуется тремя величинами: расстояние, скорость, время (3 столбца таблицы).

2. В задаче 3 процесса: движение скорого, пассажирского и товарного поездов (3 строчки таблицы).

Можно составить "скелет" таблицы.

Величины

Процессы

Расстояние (км)

Скорость (км/ч)

Время (ч)

Скорый поезд

с

с

с

Пассажирский поезд

с

с

с

Товарный поезд

с

с

с

3. Заполняем таблицу в соответствии с условиями задачи

4. Вводим неизвестные величины: x, км/ч - скорость товарного поезда, y, ч - время движения скорого поезда.

5. Составим "модель".

(x+50)y = 8/5 x(y+1)

8/5 x(y+1) = x(y+4)

6. Решаем эту систему. Из первого уравнения находим у. Из второго уравнения находим х.



Решение задачи (чистовик).

Пусть х, км/ч - скорость товарного поезда (х>0), у, ч - время движения скорого поезда (у>0).

Составляем таблицу.

Величины

Процессы

Расстояние (км)

Скорость (км/ч)

Время (ч)

Скорый поезд

(х+50)у

х+50 ?

у

Пассажирский поезд

8/5 х(у+1)

8/5 х

у+1

Товарный поезд

х(у+4)

х ?

у+4

По условию задачи поезда прошли одно и то же расстояние. Получаем систему уравнений

8/5 х(у+1) = х(у+4)

(х+50)у = х(у+4).

По условию задачи х>0, тогда

8(у+1) = 5(у+4)

(х+50)у = х(у+4),

3у = 12

(х+50)у = х(у+4),

у = 4

х+50 = 2х,

у = 4

х = 50.

Полученные значения неизвестных удовлетворяют условию х>0, у>0, значит удовлетворяют условию задачи.

50 км/ч - скорость товарного поезда.

50+50 = 100 (км/ч) - скорость скорого поезда.

Проверка по условию задачи.

50 км/ч - скорость товарного поезда,

4+4 = 8 (ч) - время движения товарного поезда.

50*8 = 400 (км) - расстояние, которое прошёл товарный поезд.

50*8/5 = 80 (км/ч) - скорость пассажирского поезда.

4+1 = 5 (ч) - время движения пассажирского поезда.

80*5 = 400 (км) - расстояние, которое прошёл пассажирский поезд.

4 ч - время движения скорого поезда.

50+50 = 100 (км/ч) - скорость скорого поезда.

100*4 = 400 (км) - расстояние, которое прошёл скорый поезд.

Каждый поезд прошёл одно и то же расстояние.

Задача решена верно.

Ответ: 50 км/ч, 100 км/ч.

Аналогично можно решать задачи "на работу", "наполнение бассейна".

Решение текстовых задач способствует, с одной стороны, закреплению на практике приобретённых умений и навыков, с другой стороны, развитию логического мышления учащихся.

Наблюдается активизация их мыслительной деятельности работы. При правильной организации работы у учащихся развивается активность, наблюдательность, находчивость, сообразительность, смекалка, развивается абстрактное мышление, умение применять теорию к решению конкретных задач.







Учебное пособие по математике

РАЗДЕЛ 4. Задачи на оптимизацию

Учебное пособие по математике

Учебное пособие по математикеУчебное пособие по математике

Важнейшим видом учебной деятельности при обучении учащихся математике является решение задач. Причем, основное внимание направлено на развитие способности учащихся, применять полученные в школе знания и умения в жизненных ситуациях. В настоящее время выявлены характерные недочеты математической подготовки школьников. К ним относятся недостаточное усвоение ряда тем, имеющих широкое практическое применение. Именно умение решать большинство из этих практических задач проверяется на ЕГЭ. Как при обучении математики сформировать ключевые компетенции?

Одним из путей формирования ключевых компетентностей является использование на уроках специальных компетентностно-ориентированных задач.

При решении компетентностно-ориентированных задач основное внимание уделяется формированию способностей учащихся использовать математические знания в разнообразных ситуациях, требующих для своего решения различных подходов, размышлений и интуиции.

Большую часть своих усилий человек тратит на поиск наилучшего, т.е. оптимального решения поставленной задачи. Как, располагая определенными ресурсами, добиваться наиболее высокого жизненного уровня, наивысшей производительности труда, наименьших потерь, максимальной прибыли, минимальной затраты времени - так ставятся вопросы, над которыми приходится думать каждому члену общества.

Математикам удалось разработать методы решения задач на отыскание наибольшего и наименьшего значения или, как их еще называют, задач на оптимизацию (от латинского "оптимум" - наилучший).Многие задачи, поиска оптимальных решений, могут быть решены только с использованием методов дифференциального исчисления. Ряд задач такого типа решается с помощью специальных методов линейного программирования, но существуют и такие экстремальные задачи, которые решаются средствами элементарной математики.

Прежде чем решать какую - либо жизненную задачу, человек старается взвесить имеющуюся у него информацию, выбрать из нее существенную. И только потом, когда станет более или менее ясно, из чего исходить и на какой результат рассчитывать, он приступает к решению задачи. Фактически это замена исходной жизненной задачи ее моделью. Разнообразие информационных аспектов в каждой такой задаче настолько велико, что бывает сложно из всего многообразия информации об изучаемом явлении или объекте выбрать наиболее существенные. В таких случаях необходимо сделать упрощающее предположение, чтобы выделить исходные данные, определить, что будет служить результатом и какова связь между исходными данными и результатом. Все это - предположения, исходные данные, результаты, связи между ними - называют моделью задачи.

Чтобы получить ответ, нужны указания, что и как делать. Такие указания часто представляются в виде алгоритма, в котором задаются математические соотношения, связывающие исходные данные и результат. В этом случае говорят о построении математической модели задачи.

Человеку часто приходиться решать задачи оптимизации в своей деятельности, в которых нужно с помощью наименьших затрат, сил, средств, материалов получить наилучший результат. Как из круглого бревна выпилить прямоугольную балку с наименьшим количеством отходов?

Каких размеров должен быть ящик при заданном расходе материала и чтобы его объем был наибольшим?В каком месте следует построить мост через речку, чтобы дорога, проходящая через него и соединяющая два города, была кратчайшей?

П.Л. Чебышев говорил, что "особенную важность имеют те методы науки, которые позволяют решать задачу, общую для всей практической деятельности человека: как располагать своими средствами для достижения наибольшей выгоды". С такими задачами в наше время приходится иметь дело представителям самых разных специальностей. Технологи - стараются так организовать производство, чтобы выпускалось как можно больше продукции. Конструкторы пытаются разработать прибор для космического корабля так, чтобы масса прибора была наименьшей. Экономисты стараются спланировать связи завода с источниками сырья так, чтобы транспортные расходы оказались минимальными, и т.д. В самых простых задачах на оптимизацию мы имеем дело с двумя величинами, одна из которых зависит от другой, причём надо найти такое значение второй величины, при котором первая принимает своё наименьшее или наибольшее (наилучшее в данных условиях) значение.

Задачи на оптимизацию решают по обычной схеме:

  • составление математической модели;

  • работа с моделью;

  • ответ на вопрос задачи.

Цель урока при изучении данной темы состоит в том, чтобы научить решать задачи на оптимизацию, используя математические модели.

Учащимся можно предложить ответить на следующие вопросы.

  1. Почему завод «Стекло» расположен вблизи железной дороги?

  2. Какие мандарины выгоднее покупать: крупные или мелкие, если толщина кожуры у них одинаковая?

  3. Какую картошку выгоднее чистить: крупную или мелкую?



Памятка по решению задач на оптимизацию

I этап. Составление математической модели.

  1. Проанализировав условия задачи, выделите оптимизируемую величину, т.е. величину, о наибольшем или наименьшем значении которой идет речь. Обозначьте ее буквой у (или S, R, V - в зависимости от содержания задачи).

  2. Одну из участвующих в задаче неизвестных величин, через которую нетрудно выразить оптимизируемую величину, примите за независимую переменную и обозначьте ее буквой х (или какой-либо другой буквой). Установите реальные границы изменения независимой переменной в соответствии с условиями задачи.

  3. Исходя из условия задачи, выразите у через х. Математическая модель задачи представляет собой функцию у=f(х) с областью определения Х, которую нашли на втором шаге.

II этап. Работа с составленной моделью.

На этом этапе для функции у=f(х), х Х найдите унаимили унаибв зависимости от того, что требуется в условии задачи. При этом используются теоретические установки, которые мы рассмотрели при определении наибольшего и наименьшего значений функции.

III этап. Ответ на вопрос задачи.

Здесь следует получить конкретный ответ на вопрос задачи, опираясь на результаты, полученные на этапе работы с моделью. Записать ответ в терминах предложенной задачи.





Учебное пособие по математике

Примеры решения задач на оптимизацию

Задача 1. Какова наибольшая площадь прямоугольного участка земли, который можно огородить куском проволоки длиной 2p?

Решение: Первый этап. Составление математической модели

  1. Выбираем независимую переменную х и выражаем через неё стороны прямоугольника. х см - длина прямоугольника, (р-х) см - ширина прямоугольника. Тогда 0<х <р;

  2. записываем функцию S(x) =x·(р-x) =рx - x2;

Второй этап. Работа с составленной моделью.

находим производную S' (x) = р-2x;

решаем уравнение р -2х = 0,

Итак, надо найти наибольшее значение функции при,

p-x=p - Следовательно, прямоугольник - квадрат со стороной и его площадь равна. Т.е. наибольшим значением площади прямоугольника будет площадь квадрата.

Третий этап. Ответ.

Задача 2. Прочность балки прямоугольного сечения пропорциональна произведению её ширины на квадрат высоты. Какое сечение должна иметь балка, вытесанная из цилиндрического бревна радиуса R, чтобы её прочность была наибольшей?

Решение:Первый этап. Составление математической модели

  1. Обозначим буквой у (оптимизируемую величину) - прочность балки;

  2. х - ширина балки (независимую переменную), 0 < x < 2R;

  3. h2=4R2-x2 - высота балки (выражается по теореме Пифагора из прямоугольного треугольника);

  4. прочность балки у = kxh2 (где коэффициент k - некоторое положительное число) значит, у = kx(4R2 - x2), где х [0; 2R].

Второй этап. Работа с составленной моделью.

Находим унаиб. Для этого воспользуемся алгоритмом нахождения наибольшего значения функции на отрезке, повторенным в начале урока.

Задача 3.Из круглого бревна радиуса R выпилить прямоугольную балку так, чтобы количество отходов было наименьшим.

Задачу можно свести к следующей: в круг радиуса R вписать прямоугольник наибольшей площади.

Решение:Первый этап. Составление математической модели

  1. Выбираем независимую переменную х и выражаем через неё стороны прямоугольника. х см - длина прямоугольника, 0 < х <2R ; см - ширина прямоугольника.;

  2. Записываем функцию S(x) =x , выражающую площадь прямоугольника.

Второй этап. Работа с составленной моделью.

Находим наибольшее значение функции S(x) = x на отрезке [0;2R].

Решаем уравнение S' (x) = 0, - = 0, 4 = 0, откуда х = R (значение х = - R не удовлетворяет условию задачи).

Вычисляем значения функции S(x) = x при х = R и на концах отрезка [0;2R]. Т.к. S(0) = S(2R) = 0, а S(R ) = 2 , то функция принимает наибольшее значение при х = R. Поскольку наибольшее значение функции

S(x) = x на отрезке [0;2R] достигается во внутренней точке отрезка, то наибольшее ее значение на интервале (0;2R).

Также достигается в точке х = R. При этом длина другой стороны прямоугольника равна = R, т.е. искомым прямоугольником служит квадрат. Наибольшее значение функции S(x) =2

Таким образом, количество отходов будет наименьшим, если в сечении балки будет квадрат.

Третий этап.Ответ.Количество отходов будет наименьшим, если в сечении балки будет квадрат со стороной R.

Задачу 3 можно решить и без использования производной.

Из неравенства Коши о среднем арифметическом и среднем геометрическом для произвольных положительных чисел,

≥ вытекают два важных утверждения.

  1. Если сумма произвольных положительных чисел, равна S, то их произведение Р = достигает наибольшего значения, равного при равенстве всех чисел.

  2. Если произведение произвольных положительных чисел, равно Р, то их сумма S = принимает наименьшее значение, равное n, при равенстве всех чисел.

Решение: Если х и у - стороны прямоугольника, S- площадь прямоугольника, то

S= ху. Т.к. прямоугольник вписан в круг, то + = 4. Следовательно,

S = х. Заметим, что Sбудет достигать наибольшего значения тогда, когда будет наибольшим = ( ). Но сумма множителей и

постоянна и равна 4, следовательно, наибольшее значение их произведения равно, а наибольшее значениеSравно 2, причем оно достигается, если

=, т.е. х = R, но тогда у = R.

Ответ.Количество отходов будет наименьшим, если в сечении балки будет квадрат со стороной R.

Рассмотрим еще один способ решения задачи 3.

Решение: Пусть величина угла между диагоналями прямоугольника равна. Тогда площадь прямоугольника равна половине произведения длин диагоналей на синус угла между ними, т.е. S( = ▪2R ▪2R = 2.

Очевидно, что наибольшее значение функции S( 2 достигается, если

= 1, т.е. =. Значит, прямоугольник является квадратом со стороной R.

Ответ. Количество отходов будет наименьшим, если в сечении балки будет квадрат со стороной R.





О решении задач на нахождение наибольшего (наименьшего) значения функции.

В настоящее время в нашей стране большое внимание уделяется вопросам повышения эффективности и качества во всех сферах производства. В этой связи особую значимость приобретает умение решать так называемые задачи на оптимизацию, которые возникают там, где необходимо выяснить как с помощью имеющихся средств достичь наилучшего результата, как получить нужный результат с наименьшей затратой средств, материалов, времени, труда и т.п.

При решении задач на нахождение наибольшего (наименьшего) значения функции надо обратить внимание на следующее:

1). Иногда приходится вводить две переменные, одна из которых обязательно длина отрезка, другая - либо длина другого отрезка, либо величина угла.

2). Часто от выбора переменной зависит и сложность решения.

3). В качестве переменной, относительно которой составляется функция для исследования, не обязательно брать искомую величину, в противном случае это может привести к более сложному решению задачи.

4). Для облегчения исследования функции p, которая положительна при всех рассматриваемых значениях переменной, полезно знать, что промежутки возрастания и убывания, точки максимума и минимума, точки, в которых функция принимает наибольшие и наименьшие значения на заданном промежутке, не изменятся, если функцию p заменить на функцию kpn, или p+a, где k, a, n - числа, причем k>0, nЄR+: у всех этих функций производная равна произведению производной функции p на положительное число.

Задача №4

В правильной четырехугольной призме сумма длин высоты и диагонали призмы равна 12. При каком угле наклона этой диагонали к плоскости основания призмы объем призмы будет наибольшим?

Решение:

Пусть BB1=x, где x>0 - необходимое условие.

В1D=12-x,

Учебное пособие по математикеB1BD: по теореме Пифагора

BD= Учебное пособие по математике=Учебное пособие по математике=Учебное пособие по математике;

Vпризмы= Учебное пособие по математикеBB1=Учебное пособие по математикеx=12(6-x)x

Учебное пособие по математике

Рассмотрим непрерывную функцию p(x)= 6x-x2 при x>0.

p'(x)=6-2x;

Найдем критические точки функции p(x).

p'(x)=0; 6-2x=0; x=3.

Исследуем критическую точку на экстремум.

Учебное пособие по математике

При 00;

При x>3 p'(x)<0.

Значит, функция p, непрерывная в точке 3, возрастает при 0Учебное пособие по математике3 и убывает при xУчебное пособие по математике3.

Следовательно, при x=3 функция p и Vпризмы=12p(x) будут иметь наибольшее значение.

Теперь найдем искомый угол a.

Так как BB1=3; B1D=9, то sin a =Учебное пособие по математике, a = arcsinУчебное пособие по математике.

Ответ: arcsinУчебное пособие по математике.





Задача №5.

В правильной пирамиде МАВСD МО - высота, МК - апофема пирамиды, МК=6Учебное пособие по математике. Найти длину МО, при которой объем пирамиды будет наибольшим.

Учебное пособие по математике

Решение.

Пусть МО=x, где x>0 - необходимое условие.

ОК2=МК2 - МО2=108-x2;

Vпир.= (2ОК)2·Учебное пособие по математике=Учебное пособие по математике(108-x2)x.

Рассмотрим непрерывную на R функцию p(x)= (108-x2)x=108-x2 при x>0.

p'(x)=108-3x2=3(36-x2);

p'(x)=0; 3(36-x2)=0;

36-x2=0;

x=-6 или x=6.

x=-6 - не удовлетворяет условию x>0.

Исследуем критическую точку x=6 на экстремум.

Учебное пособие по математике

При 00;

при x>6 p'(x)<0;

Значит, функция p, непрерывная в точке 6, возрастает при 0 и убывает при x?6, следовательно, имеет наибольшее значение в точке 6. Поэтому Vпир.=Учебное пособие по математике при x=6 имеет наибольшее значение.

Ответ: 6.

Задача №6.

Учебное пособие по математике

В правильной пирамиде MABCD МО - высота пирамиды, МК - апофема, МК+МО=6, МКУчебное пособие по математике[4,5]. Найдите длину МК, при которой площадь боковой поверхности пирамиды будет наименьшей.

Решение:

Пусть МК=x, тогда МО=6-x, где 4Учебное пособие по математикеxУчебное пособие по математике5. DK=MO;

Учебное пособие по математике;

Sбок.Учебное пособие по математике, где x>0.

Рассмотрим непрерывную функцию p(x)=x3-3x2, при x>0.

p'(x)=3x2-6x,

p'(x)=0 при x=2.

Исследуем критическую точку x=2 на экстремум.

Учебное пособие по математике

При x>2 p'(x)>0, значит, функция p(x) при 4Учебное пособие по математикеxУчебное пособие по математике5 возрастает.

Следовательно, при x=4 функция p на отрезке [4,5], значит, и Sбок.Учебное пособие по математике имеют наименьшие значения.

Ответ: 4.

Поиск рационального решения задачи на экстремум.

При решении задачи на экстремум учащиеся нередко испытывают трудности в составлении аналитической записи функции, описывающей условие задачи. Причиной этому часто бывает нерациональный выбор независимой переменной. Ее желательно выбрать так, чтобы более коротким путем получить аналитическое выражение искомой функции и чтобы это выражение было по возможности более простым.

Однако, при удачном выборе аргумента функции удается сократить вычисления и упростить решение задачи.

Задача 7.

В окружность радиуса R вписана трапеция АВСD, основание АВ которой является диаметром окружности.

Какова должна быть длина боковой стороны трапеции, чтобы трапеция имела наибольшую площадь?

Решение.

Учебное пособие по математике

Sтр.= Учебное пособие по математике, где DH - высота трапеции или по формуле: Sтр.=BH·DH, так как трапеция равнобочная и Учебное пособие по математике.

Первый способ.

В задаче требуется найти длину боковой стороны трапеции, при которой площадь трапеции будет наибольшей. Ее можно принять за независимую переменную, затем через нее и радиус окружности R выразить площадь трапеции.

Учащиеся обычно так и поступают. Пусть AD=x; ?ABD - прямоугольный, поэтому AD2=AB·AH, откуда Учебное пособие по математике.По теореме Пифагора

Учебное пособие по математике; а так как Учебное пособие по математике, то Учебное пособие по математике.

По смыслу задачи 0Учебное пособие по математике.

При Учебное пособие по математике трапеция вырождается в равнобедренный треугольник.

Рассмотрим функцию Учебное пособие по математике. Найдем производную p'(x). После очевидных сокращений получим: Учебное пособие по математике.

В промежутке между 0 и Учебное пособие по математике производная обращается в нуль лишь в точке x=R, меняя при этом знак с плюса на минус. Значит, при x=R p(x) имеет наибольшее значение. Таким образом, площадь трапеции будет наибольшей при AD=R. Легко заметить, что искомая трапеция имеет форму половины правильного шестиугольника. Ее площадь равна Учебное пособие по математике.

Второй способ.

Обозначим через x высоту DH трапеции. Из прямоугольного треугольника ODH (О - центр окружности) находим: Учебное пособие по математике, значит Учебное пособие по математике, и получим Учебное пособие по математике, где 0

Третий способ.

Пусть BH=x. Тогда AH=2R-x. Согласно свойству высоты прямоугольного треугольника ABD имеем:

Учебное пособие по математике и, следовательно,

Учебное пособие по математике, R.

Производная функции Учебное пособие по математике находится проще, чем при решении задачи первым и вторым способом. Более того, при таком выборе независимой переменной задачу можно решить и без использования производной.

Заметим, что Учебное пособие по математике.

Первая часть равенства представляет собой произведение переменных, сумма которых постоянна и равна 6R. Следовательно, это произведение принимает наибольшее значение в случае их равенства, т.е. Учебное пособие по математике, откуда Учебное пособие по математике. При этом Учебное пособие по математике и AD=OD=R.

Итак, третий способ выбора независимой переменной предпочтительнее первых двух.

Однако, в качестве независимой переменной можно выбрать и величину угла BAD.

Четвертый способ.

Пусть Учебное пособие по математикеУчебное пособие по математике. Тогда Учебное пособие по математике, Учебное пособие по математике, Учебное пособие по математике.

Далее находим: Учебное пособие по математике. S'=0 при Учебное пособие по математике, т.е. при Учебное пособие по математике. Остается сравнить значения функции Ы в критической точке со значениями на концах промежутка [45o; 90o].

Пятый способ.

Введем независимую переменную: Учебное пособие по математике. Площадь трапеции равна сумме площадей трех треугольников: AOD, BOC и COD. Следовательно, Учебное пособие по математике, 0oo.

Таким образом, задача легко сводится к нахождению наибольшего значения функции Учебное пособие по математике. Ее производная Учебное пособие по математике.

Критические точки получим, решив уравнение Учебное пособие по математике.

После разбора различных способов решения задачи учащимся можно предложить обобщение этой задачи.

Задачи для самостоятельного решения

Задача 1

Найти высоту и радиус основания прямого кругового цилиндра наибольшего объема, вписанного в прямой круговой конус с радиусом основания R и высотой 2R так, что основание цилиндра лежит в плоскости основания конуса.

Учебное пособие по математике

Ответ: Учебное пособие по математике.

Задача 2

Сумма катетов прямоугольного треугольника 9 см. При вращении треугольника вокруг одного из катетов образуется конус максимального объема V. Найти Sбок. этого конуса.

Ответ: Учебное пособие по математикесм2.

Задача 3.

Найти высоту конуса наименьшего объема, описанного около полушара радиуса Учебное пособие по математике так, чтобы центр основания совпадал с центром шара.

Ответ: 3.

Задача 4.

Правильная треугольная призма имеет объем 16 дм3. Найти длину стороны основания призмы с наименьшей полной поверхностью.

Ответ: 4.

Задача 5.

Периметр боковой грани правильной шестиугольной призмы равен 6 см. Найти площадь боковой поверхности призмы, имеющей наибольший объем, если сторона основания не больше высоты призмы.

Ответ: 13,5 см2.

Тестовые задания

1. Найдите наименьшее значение функции Учебное пособие по математике при условии Учебное пособие по математике.

2. Найдите наименьшее значение функции Учебное пособие по математике при условии Учебное пособие по математике.

3. Найдите решения неравенства Учебное пособие по математике, при которых функция Учебное пособие по математикепринимает наибольшее значение.

4. Найдите наименьшее значение функции Учебное пособие по математике при условии Учебное пособие по математике.

5. Найдите наименьшее значение функции Учебное пособие по математике при условии Учебное пособие по математике.

6. Найдите решения неравенства Учебное пособие по математике, при которых функция Учебное пособие по математикепринимает наименьшее значение.

Ответы С1

1. Учебное пособие по математике

2. Учебное пособие по математике

3. - 3

Учебное пособие по математике4. Учебное пособие по математике

5. Учебное пособие по математике



РАЗДЕЛ 5. Комбинаторика

Учебное пособие по математике

КОМБИНАТОРИКА - раздел математики, в котором изучаются вопросы о том, сколько различных комбинаций, подчиненных тем или иным условиям, можно составить из заданных объектов.

Комбинаторику можно рассматривать как введение в теорию вероятностей, поскольку методы комбинаторики используются для решения многих вероятностных задач, в которых речь идет о подсчете числа возможных исходов и числа благоприятных исходов в различных конкретных случаях.

Выбором объектов и расположением их в том или ином порядке приходится заниматься чуть ли не во всех областях человеческой деятельности.

С аналогичными задачами, получившими название комбинаторных, люди сталкивались в глубокой древности. Уже несколько тысячелетий назад в Древнем Китае увлеклись составлением магических квадратов, в которых заданные числа располагались так, что их сумма по всем горизонталям, вертикалям и главным диагоналям была одной и той же. В Древней Греции подсчитывали число различных комбинаций длинных и коротких слов в стихотворных размерах, занимались теорией фигурных чисел, изучали фигуры, которые можно составить из частей особым образом разрезанного квадрата и т.д.

Комбинаторные задачи возникли и в связи с такими играми, как шашки, шахматы, домино, карты, кости и т.д.

Комбинаторика становится наукой лишь в 18 веке - в период, когда возникла теория вероятностей. Чтобы решать теоретико-вероятностные задачи, нужно было уметь подсчитывать число различных комбинаций, подчиненных тем или иным условиям. После первых работ, выполненных в 18 веке итальянским ученым Дж. Кардано, Н. Тартальей, и Г. Галилеем, такие задачи изучали французские математики Б. Паскаль и П. Ферма. Первым рассматривал комбинаторику как самостоятельную ветвь науки немецкий философ и математик Г. Лейбниц, опубликовавший в 1666 году работу " Об искусстве комбинаторики", в которой впервые появляется сам термин "комбинаторный".

Замечательные достижения в области комбинаторики принадлежат Л.Эйлеру. Комбинаторными задачами интересовались и математики, занимавшиеся составлением и разгадыванием шифров, изучением древних письменностей. Теперь комбинаторика находит применение во всех областях науки и техники: в биологии, где она применяется для изучения состава белков и ДНК, в химии, в механике и т.д.

По мере развития комбинаторики выяснилось, что, несмотря на внешнее различие изучаемых ею вопросов, многие из них имеют одно и то же математическое содержание и сводятся к задачам о конечных множествах и их подмножествах. Постепенно выяснилось несколько основных типов задач, к которым сводится большинство комбинаторных проблем. Важную область комбинаторики составляет теория перечислений. С ее помощью можно пересчитать число решений различных комбинаторных задач.

Задачи по темам «Комбинаторика» с решениями

Задача «Замок секретом». Для запирания сейфов, ячеек, специальным образом, оборудованных помещений для хранения секретной документации, используются замки, которые открываются только тогда, когда на индикаторе набрано слово «СЛОВО» (некоторая последовательность букв). Пусть слово состоит из четырёх букв и на каждой позиции может оказаться любая из 32 букв. Сколько неудачных попыток может сделать грабитель, решивший открыть замок?

Решение: На первую позицию можно поместить любую из 32 букв, на вторую позицию можно так же поместить любую из 32 букв. Поэтому всего имеем Учебное пособие по математике=1024 комбинаций из двух букв. Рассуждая аналогично, получаем, что всего имеется Учебное пособие по математике=Учебное пособие по математике=1.048.576 комбинаций из 4-х букв, среди которых только одна комбинация дает искомое "СЛОВО". Поскольку уже после первой неудачной попытки раздается сигнал тревоги, шанс грабителя на успех не велик.

В разделе математики, который назван комбинаторикой, решаются задачи, связанные с рассмотрением множеств и составлений различных комбинаций из элементов этих множеств. Полученные комбинации удовлетворяют различным условиям. В зависимости от правил составления можно выделить три такие комбинации: перестановка, размещение, сочетание.





Формула включений и исключений

Пусть имеется N предметов, каждый из которых может обладать свойствами aУчебное пособие по математике,aУчебное пособие по математике,…aУчебное пособие по математике.При этом каждый предмет может обладать одним или несколькими свойствами, а может не обладать ни одним из этих свойств. Обозначим через N(aУчебное пособие по математике) количество предметов, образующих свойствами Учебное пособие по математике Если необходимо отметить предмет, не обладающим некоторым свойством, то это свойство пишется с чертой сверху. N (Учебное пособие по математикеУчебное пособие по математике).число предметов, не обладающих ни одним из указанных свойств, обозначается N (Учебное пособие по математике).Общий закон состоит в том, что:

N (Учебное пособие по математике) =

=N-N(Учебное пособие по математике)-N(Учебное пособие по математике)-…-N(Учебное пособие по математике)+N(Учебное пособие по математике)+N(Учебное пособие по математике)+…+N(Учебное пособие по математике)+N(Учебное пособие по математике)+…+N(Учебное пособие по математике)+…+ +(-1)Учебное пособие по математикеУчебное пособие по математике. (Объединить в одну строку. Это одна формула!).

Данную формулу назовем формулой включений и исключений - сначала исключаются все предметы обладающие хотя бы одним из свойствУчебное пособие по математике,потом включаются предметы, обладающие по крайней мере двумя из этих свойств, исключаются по крайней мере три и т. д.

Задача «Анализ отчета». Староста класса дал следующие сведения об учениках: «В классе 45 учеников, 25 мальчиков. Учатся на «4» и «5» 30 учеников, в том числе 16 мальчиков. Спортом занимаются 28 учеников, среди которых 18 мальчиков и 17 школьников, учащихся на «4» и «5». И в то же время занимаются спортом 15 мальчиков». Через несколько дней классный руководитель обнаружил ошибку в сведениях.

Решение. Обозначим через «м» мальчиков, «у» - хорошую успеваемость, «с» - увлечение спортом. Подсчитаем, сколько девочек не занимается спортом и получает «3», т. е. N(Учебное пособие по математике Учебное пособие по математике Учебное пособие по математике).

По условию задачи имеем:

N(м)=25, N(у)=30, N©=28

N(м у)=16, N(м с)=18

N(у с)=17, N(м у с)=15

Значит, по формуле включений и исключений имеем:

N(Учебное пособие по математикеУчебное пособие по математикеУчебное пособие по математике)=45-25-30-28+16+18+17-15=-2

Но отрицательный ответ быть не может. Поэтому в данных сведениях есть внутреннее противоречие, они не верны.

Определение факториала

Произведение всех натуральных чисел от 1 до n, включительно, называется n-факториалом.Учебное пособие по математике; 3!= Учебное пособие по математике=6

Определение: Комбинацией из n элементов, которые отличаются друг от друга только порядком элементов, называется перестановками.

Перестановки обозначаются символом Учебное пособие по математике, где n-число элементов, входящих в каждую перестановку.

Учебное пособие по математике или Учебное пособие по математике

Задача 1. Сколько шестизначных чисел, кратных 5, можно составить из цифр: 0,1,2,3,4,5, при условии, что в цифрах нет одинаковых цифр?

Решение. Число, кратное 5, должно оканчиваться либо на «0», либо на «5». Если последняя цифра - «0», то остальные 5 цифр можно располагать в любом порядке. Следовательно, шестизначных чисел, которые заканчиваются цифрой «0» столько, сколько можно сделать Учебное пособие по математике.

Если последней цифрой числа является «5», то остальные пять цифр можно расположить Учебное пособие по математике способами. Но из этих способов не действительны те, где на первом месте «0».

Значит, количество чисел, кратных 5 и оканчивающихся на «5», равноУчебное пособие по математике.

Всего имеется Учебное пособие по математике шестизначных чисел кратных 5.





Задачи по теме «Размещение, перестановки, сочетания»

Определение: Комбинации из m элементов по n элементов, которые отличаются друг от друга или самими элементами, или порядком элементов, называется размещением.

Формула размещения: Учебное пособие по математике

Задача 2.

Учебное пособие по математике

Учебное пособие по математике

Учебное пособие по математике

Учебное пособие по математике

Учебное пособие по математике

Задача 2. Вычислить в факториальной форме:

Учебное пособие по математике

Учебное пособие по математике

Задача 3. В седьмом классе изучают 14 предметов. Сколькими способами можно составить расписание на субботу, если в этот день должно быть 5 уроков разных?

Решение. Учебное пособие по математике

Определение: Сочетаниями называются все возможные комбинации из m элементов по n, которые отличаются друг от друга по крайней мере хотя бы одним элементом:

Учебное пособие по математике

Задача 4. 12 рабочих разбиты на три бригады по 4 человека в каждой. Сколько может быть различных составов бригад?

Решение.

Учебное пособие по математике

Задача 5. Собрание из 80 человек выбирает представителя, секретаря и 3-х членов редакционной комиссии: сколькими способами можно это сделать?

Решение.

Учебное пособие по математике

Следовательно, Учебное пособие по математике

Задача 6. Сколько различных дробей можно составить из чисел 3, 5, 7, 11, 13, 17 так, чтобы в каждую дробь входило два числа?

Решение. Число дробей определяется числом размещением

Учебное пособие по математике







Подбор и решение задач на основные формулы комбинаторики и Бином Ньютона

Бином Ньютона. Натуральная степень суммы двух величин вычисляется по формуле:

Учебное пособие по математике

Учебное пособие по математике- биноминальные коэффициенты.

Задача 7. Возвести в шестую степень двучлен Учебное пособие по математике

Решение. Пусть Учебное пособие по математике

Учебное пособие по математике =Учебное пособие по математикеУчебное пособие по математике

Задача 8. Найти четвертый член разложения степени бинома.

(Учебное пособие по математике

Учебное пособие по математике

; b=Учебное пособие по математике , тогда Учебное пособие по математике

Задача 9. Учебное пособие по математике

Учебное пособие по математике+Учебное пособие по математике

Задача 10. Сколькими способами из восьми человек можно избрать комиссию, состоящую из пяти членов?

Решение. Для решения задачи необходимо использовать формулу для сочетания элементов, т. к. здесь не имеет значения порядок элементов в выборе. Запишем формулу для сочетаний.

Учебное пособие по математике

Задача 11. Сколькими способами можно расставить на полке семь книг, если две определенные книги должны стоять рядом.

Решение. Книги, которые должны стоять рядом, считаем за одну книгу. Тогда нужно расставит 6 книг по шести местам. Применяем формулу перестановок, получаем Учебное пособие по математике

Мы учли перестановки шести книг, не учитывая порядок внутри тех книг, которые посчитали за одну. А т. к. две книги по двум местам можно разместить только двумя способами (Учебное пособие по математике) ,то получаем (Учебное пособие по математике)

Задача 12. Сколько четырехбуквенных слов можно образовать из букв слова "САПФИР"? Сколько среди них таких, которые не содержат буква «р»? Сколько таких, которые начинаются с буквы «с» и оканчиваются буквой «р»?

Решение. 1). Из шести букв составляются четырехбуквенные слова, причем порядок букв важен для образования новых слов. Поэтому используется формула для размещения:

Учебное пособие по математике

2). Необходимо исключить букву «р» раз рассматривается количество слов, не содержащих данную букву:

Учебное пособие по математике

3). На первое место поставить букву «c» можно только одним способом. На последнее место поставить букву «р» можно тоже только одним способом, остаются 4 буквы, которые необходимо разместить по двум местам:

Учебное пособие по математике

Задача 13. Сколько пятибуквенных слов, каждое из которых состоит из трех согласных и двух гласных, можно образовать из букв слова «УРАВНЕНИЕ!»?

Решение. В слове "УРАВНЕНИЕ" 3 согласных и 4 гласных буквы, чтобы посчитать количество требуемых пятибуквенных слов, необходимо посчитать количество сочетаний трех согласных из трех заданных и двух гласных из четырех заданных Учебное пособие по математикеи Учебное пособие по математике. После того, как 5 букв выбраны, необходимо посчитать все возможные перестановки этих букв: Учебное пособие по математике

Учебное пособие по математике

Задачи для самостоятельного решения

Задача 1:

В магазине «Все для чая» есть 5 разных чашек и 3 разных блюдца. Сколькими способами можно купить чашку с блюдцем?

Задача 2:

В магазине «Все для чая» есть еще 4 чайные ложки. Сколькими способами можно купить комплект из чашки, блюдца и ложки?

Задача 3:

В Стране Чудес есть три города: А, Б и В. Из города А в город Б ведет 6 дорог, а из города Б в город В - 4 дороги. Сколькими способами можно проехать от А до В?

Задача 4:

В Стране Чудес есть четыре города: А, Б и В и Г. Из города А в город Б ведет 6 дорог, а из города Б в город В - 4 дороги, Из города А в город Г - две дороги, и из города Г в город В - тоже две дороги. Сколькими способами можно проехать от А до В?

Задача 5:

В магазине «Все для чая» по-прежнему продается 5 чашек, 3 блюдца и 4 чайные ложки. Сколькими способами можно купить два предмета с разными названиями?

Задача 6:

Назовем натуральное число «симпатичным» , если в его записи встречаются только нечетные цифры. Сколько существует 4-значных «симпатичных» чисел?

Задача 7:

Монету бросают трижды. Сколько разных последовательностей орлов и решек можно при этом получить?

Задача 8:

Каждую клетку квадратной таблицы 2 × 2 можно покрасить в черный или белый цвет. Сколько существует различных раскрасок этой таблицы?

Задача 9:

Сколькими способами можно заполнить одну карточку в лотерее «Спорт-про-г-ноз»? (В этой лотерее нужно предсказать итог тринадцати спортивных матчей. Итог каждого матча - победа одной из команд либо ничья; счет роли не играет).

Задача 10:

Алфавит племени Мумбо-Юмбо состоит из трех букв А, Б и В. Словом является любая последовательность, состоящая не более, чем из 4 букв. Сколько слов в языке племени Мумбо-Юмбо? Указание. Сосчитайте отдельно количества одно-, двух-, трех- и четырехбуквенных слов.

Задача 11:

В футбольной команде (11 человек) нужно выбрать капитана и его заместителя. Сколькими способами это можно сделать?

Задача 12:

Сколькими способами можно сделать трехцветный флаг с горизонтальными полосами одинаковой ширины, если имеется материя шести различных цветов?

Задача 13:

Сколькими способами можно поставить на шахматную доску белую и черную ладьи так, чтобы они не били друг друга?

Задача 14:

Сколькими способами можно поставить на шахматную доску белого и черного королей так, чтобы получилась допустимая правилами игры позиция?

Задача 15:

Сколько существует трехзначных чисел, в записи которых цифры 1, 2, 3 встречаются ровно по одному разу?

Задача 16:

Сколькими способами можно выложить в ряд красный, черный, синий и зеленый шарики?

Задача 17: Слово - любая конечная последовательность букв русского алфавита. Выясните, сколько различных слов сожно составить из слов

а) «ВЕКТОР»;

б) «ЛИНИЯ»;

в) «ПАРАБОЛА»;

г) «БИССЕКТРИСА»;

д) «МАТЕМАТИКА»;

Задача 22:

В стране 20 городов, каждые два из которых соединены авиалинией. Сколько авиалиний в этой стране?

Задача 23:

Сколько диагоналей в выпуклом n-угольнике?

Задача 24:

Бусы - это кольцо, на которое нанизаны бусины. Бусы можно поворачивать, но не переворачивать. Сколько различных бус можно сделать из 13 разноцветных бусин?

Задача 25:

Предположим теперь, что бусы можно и переворачивать. Сколько тогда различных бус можно сделать из 13 разноцветных бусин?

Задача 26:

Сколько существует 6-значных чисел, в записи которых есть хотя бы одна четная цифра?

Задача 27:

В алфавите племени Бум-Бум шесть букв. Словом является любая последовательность из шести букв, в которой есть хотя бы две одинаковые буквы. Сколько слов в языке племени Бум-Бум?

Задача 28:

В киоске «Союзпечать» продаются 5 видов конвертов и 4 вида марок. Сколькими способами можно купить конверт с маркой?

Задача 29:

Сколькими способами можно выбрать гласную и согласную буквы из слова «КРУЖОК»?

Задача 30:

На доске написаны 7 существительных, 5 глаголов и 2 прилагательных. Для предложения нужно выбрать по одному слову каждой из этих частей речи. Сколькими способами это можно сделать?

Задача 31:

У двух начинающих коллекционеров по 20 марок и по 10 значков. Честным обменом называется обмен одной марки на одну марку или одного значка на один значок. Сколькими способами коллекционеры могут осуществить честный обмен?

Задача 32:

Сколько существует 6-значных чисел, все цифры которых имеют одинаковую четность?

Задача 33:

Надо послать 6 срочных писем. Сколькими способами это можно сделать, если для передачи писем можно использовать трех курьеров и каждое письмо можно дать любому из курьеров?

Задача 34:

Сколькими способами из полной колоды (52 карты) можно выбрать 4 карты разных мастей и достоинств?

Задача 35:

На полке стоят 5 книг. Сколькими способами можно выложить в стопку несколько из них (стопка может состоять и из одной книги)?

Задача 36:

Сколькими способами можно поставить 8 ладей на шахматную доску так, чтобы они не били друг друга?

Задача 37:

На танцплощадке собрались N юношей и N девушек. Сколькими способами они могут разбиться на пары для участия в очередном танце?

Задача 38:

Чемпионат России по шахматам проводится в один круг. Сколько играется партий, если участвуют 18 шахматистов?

Задача 39:

Сколькими способами можно поставить на шахматную доску так, чтобы они не били друг друга а) две ладьи; б) двух королей; в) двух слонов; г) двух коней; д) двух ферзей?

Задача 40:

У мамы два яблока, три груши и четыре апельсина. Каждый день в течение девяти дней подряд она дает сыну один из оставшихся фруктов. Сколькими способами это может быть сделано?

Задача 41:

Сколькими способами можно поселить 7 студентов в три комнаты: одноместную, двухместную и четырехместную?

Задача 42:

Сколькими способами можно расставить на первой горизонтали шахматной доски комплект белых фигур (король, ферзь, две ладьи, два слона и два коня)?

Задача 43:

Сколько слов можно составить из пяти букв А и не более чем из трех букв Б?

Задача 44:

Сколько существует 10-значных чисел, в которых имеется хотя бы две одинакоые цифры?

Задача 45:

Каких 7-значных чисел больше: тех, в записи которых есть 1, или остальных?

Задача 46:

Кубик бросают трижды. Среди всех возможных последовательностей результатов есть такие, в которых хотя бы один раз встречается шестерка. Сколько их?

Учебное пособие по математике

РАЗДЕЛ 6. Элементы теории вероятностей и математической статистики

Учебное пособие по математике


Один из важнейших аспектов модернизации содержания математического образования состоит во включении в программы элементов статистики, теории вероятностей и комбинаторики. Это обусловлено ролью, которую играют вероятностно-статистические знания в общеобразовательной подготовке современного человека. Без минимального вероятностно-статистической грамотности трудно адекватно воспринимать социальную, политическую, экономическую информацию и принимать на ее основе обоснованные решения. Современные науки, такие как физика, химия, биология, весь комплекс социально-экономических наук построены и развиваются на вероятностно-статистической базе.

Включение элементов статистики, теории вероятностей и комбинаторики в государственный стандарт основного общего образования потребовало более тщательного осмысления методики преподавания этих разделов математики.

  1. Начало комбинаторики и вычисление вероятностей при помощи подсчета числа благоприятных случаев

  2. Операции над событиями, теоремы сложения вероятностей, условные вероятности, независимые события

  3. Независимые повторные испытания с постоянной вероятностью, теорема Бернулли. Беседа о законе больших чисел и его роли в развитии различных областей науки и практической деятельности

  4. Математическое ожидание. Дисперсия и закон больших чисел.

Важным моментом при моделировании содержания и процесса обучения математики с учетом включения в него статистического материала выступает принцип межпредметных связей.

В результате студент должен

знать/понимать

  • вероятностный характер многих закономерностей окружающего мира; примеры статистических закономерностей и выводов;

уметь

  • проводить несложные доказательства, получать простейшие следствия из известных или ранее полученных утверждений, оценивать логическую правильность рассуждений, использовать примеры для иллюстрации и контрпримеры для опровержения утверждений;

  • извлекать информацию, представленную в таблицах, на диаграммах, графиках; составлять таблицы, строить диаграммы и графики;

  • решать комбинаторные задачи путем систематического перебора возможных вариантов, а также с использованием правила умножения;

  • вычислять средние значения результатов измерений;

  • находить частоту события, используя собственные наблюдения и готовые статистические данные;

  • находить вероятности случайных событий в простейших случаях;

использовать приобретенные знания и умения в практической деятельности и повседневной жизни для:

  • выстраивания аргументации при доказательстве (в форме монолога и диалога);

  • распознавания логически некорректных рассуждений;

  • записи математических утверждений, доказательств;

  • анализа реальных числовых данных, представленных в виде диаграмм, графиков, таблиц;

  • решения практических задач в повседневной и профессиональной деятельности с использованием действий с числами, процентов, длин, площадей, объемов, времени, скорости;

  • решения учебных и практических задач, требующих систематического перебора вариантов;

  • сравнения шансов наступления случайных событий, оценки вероятности случайного события в практических ситуациях, сопоставления модели с реальной ситуацией;

  • понимания статистических утверждений.









Как решать задачи на вероятность?

Если вас интересует вопрос заголовка, вы наверняка студент или школьник, столкнувшийся с новым для себя предметом. Задачи теории вероятностей сейчас решают и школьники пятых классов продвинутых школ, и старшеклассники перед ЕГЭ, и студенты буквально всех специальностей - от географов до математиков. Что же это за предмет такой, и как к нему подойти?

Вероятность. Что это?

Теория вероятностей, как следует из названия, имеет дело с вероятностями. Нас окружают множество вещей и явлений, о которых, как бы ни была развита наука, нельзя сделать точных прогнозов. Мы не знаем, какую карту вытянем из колоды наугад или сколько дней в мае будет идти дождь, но, имея некоторую дополнительную информацию, можем строить прогнозы и вычислять вероятности этих случайных событий.

Алгоритм решения типовых задач на нахождение вероятности

  • Внимательно прочитать задачу и понять, что именно происходит (что из какого ящика вытаскивается, что где лежало, сколько приборов работает и т.п.)

  • Найти основной вопрос задачи вроде "вычислить вероятность того, что ..." и вот это многоточие записать в виде события, вероятность которого надо найти.

  • Событие записано. Теперь надо понять, к какой "схеме" теории вероятностей относится задача, чтобы правильно выбрать формулы для решения. Ответьте на тестовые вопросы типа:

    • происходит одно испытание (например, выбрасывание двух костей) или несколько (например, проверка 10 приборов);

    • если испытаний несколько, зависимы ли результаты одного от других (зависимость или независимость событий);

    • событие происходит в единственной ситуации или задача говорит о нескольких возможных гипотезах (например, шар вынимается из любого ящика из трех, или из конкретного).

Чем больше опыт решения задач, тем легче будет определить, какие формулы подходят.

  • Выбрана формула (или несколько) для решения. Записываем все данные задачи и подставляем в данную формулу.

  • вероятность найдена.

Как решать задачи: классическая вероятность

Пример 1. В группе из 30 студентов на контрольной работе 6 студентов получили «5», 10 студентов - «4», 9 студентов - «3», остальные - «2». Найти вероятность того, что 3 студента, вызванные к доске, получили по контрольной работе «2».

Начинаем решение по пунктам

В задаче речь идет о выборе 3 студентов из группы, которые удовлетворяют определенным условиям.

  • Вводим основное событие X = (Все 3 студента, вызванные к доске, получили по контрольной работе «2»).


  • Так как в задаче происходит только одно испытание и оно связано с отбором/выбором по определенному условию, речь идет о классическом определении вероятности. Запишем формулу: P=m/n, где m - число исходов, благоприятствующих осуществлению события X, а n - число всех равновозможных элементарных исходов.


  • Теперь необходимо найти значения m и n для этой задачи. Сначала найдем число всех возможных исходов - число способов выбрать 3 студентов из 30. Так как порядок выбора не имеет значения, это число сочетаний из 30 по 3:

n=C330=30!3!27!=28⋅29⋅301⋅2⋅3=4060.

Найдем число способов вызвать только студентов, получивших "2". Всего таких студентов было 30−6−10−9=5 человек, поэтому

m=C35=5!3!2!=4⋅51⋅2=10.

  • Получаем вероятность:

P(X)=mn=104060=0,002.

Задача решена.

Решенные задачи на классическое определение вероятности.

Как решать задачи: формула Бернулли

Пример 2.Какова вероятность того, что при 8 бросаниях монеты герб выпадет 5 раз?

Снова по схеме решения задач на вероятность рассматриваем данную задачу:

  • В задаче идет речь о серии одинаковых испытаний - бросаний монеты.



  • Вводим основное событие X = (При 8 бросаниях монеты герб выпадет 5 раз).


  • Так как в задаче происходит несколько испытаний, и вероятность появления события (герба) одинакова в каждом испытании, речь идет о схеме Бернулли. Запишем формулу Бернулли, которая описывает вероятность того, что из n бросков монет герб выпадет ровно k раз:

Pn(k)=Ckn⋅pk⋅(1−p)n−k.

  • Записываем данные из условия задачи: n=8,p=0,5 (вероятность выпадения герба в каждом броске равна 0,5) и k=5


  • Подставляем и получаем вероятность:

P(X)=P8(5)=C58⋅0,55⋅(1−0,5)8−5=8!5!3!⋅0,58=6⋅7⋅81⋅2⋅3⋅0,58=0,219.

Задача решена.

Учебное пособие по математике



Задачи для самостоятельного решения

№1. Решить задачу.

1. Набирая номер телефона, абонент забыл последние три цифры, и помня лишь, что эти цифры различны, набрал их наудачу. Найти вероятность того, что набраны нужные цифры.

2. В цехе работают 6 мужчин и 4 женщины. По табельным номерам наудачу отобраны 7 человек. Найти вероятность того, что среди отобранных лиц окажутся 3 женщины.

3. В группе 12 студентов, среди которых 8 отличников. По списку наудачу отобраны 9 студентов. Найти вероятность того, что среди отобранных студентов 5 отличников.

4. Собрание, на котором присутствует 25 человек, в т.ч. 5 женщин, выбирает делегацию из 3 человек. Считая, что каждый из присутствующих с одинаковой вероятностью должна быть избран, найти вероятность того, что в делегацию войдут 2 женщины и 1 мужчина.

5. На полке наудачу расставляют 10 книᴦ. Найти вероятность того, что 3 определённые книги окажутся рядом.

6. Бросают 4 игральные кости. Найти вероятность того, что на всех выпадет одинаковое число очков.

7. Группа из 10 мужчин и 10 женщин делится случайным образом на две равные части. Найти вероятность того, что в каждой части мужчин и женщин одинаково.

8. В зале 50 мест. Найти вероятность того, что из 10 человек 5 займут определенные места͵ если места занимаются ими случайным образом.

9. Для производственной практики на 30 студентов предоставлено 15 мест в Рязани, 8 в Тамбове и 7 в Воронеже. Какова вероятность того, что два определенных студента попадут на практику в один и тот же город?

10. В партии из 10 изделий имеется 4 бракованных. Наугад выбирают 5 изделий. Определить вероятность того, что среди этих 5 изделий окажется 3 бракованных.

11. три стрелка стреляют по цели. Вероятность попадания в цель для первого стрелка равна 0,75; для второго 0,8; для третьего 0,9. Найти вероятность того, что˸ 1) все три стрелка попадут в цель; 2) все трое промахнутся; 3)только один стрелок попадёт в цель; 4)хотя бы один стрелок попадёт в цель.

12. В первом ящике 6 белых и 4 чёрных шара, во втором-7 белых и 3 чёрных. Из каждого ящика наугад вынимают по одному шару. Чему равна вероятность того, что вынутые шары разного цвета.

13. На двух станках производится одинаковые детали. Вероятность того, что деталь стандартная, для первого станка равна 0,8, для второго 0,9. Производительность второго станка втрое больше, чем первого. Найти вероятность того, что взятая наудачу деталь окажется стандартной.

14. На пяти карточках написано по одной цифре из набора 1, 2, 3, 4, 5. Наугад выбирают одну за другой две карточки. Какова вероятность того, что число на второй карточке будет больше, чем на первой?

15. Из партии, в которой 20 деталей без дефектов и 5 с дефектами, берут наудачу 3 детали. Чему равна вероятность того, что˸ 1) все три детали без дефектов; 2) по крайней мере одна деталь без дефектов?

16. Слово ʼʼкаретаʼʼ, составленное из букв-кубиков, рассыпано на отдельные буквы, которые затем сложены в коробке. Из коробки наугад извлекают буквы одну за другой. Какова вероятность получить при таком извлечении слово ʼʼракетаʼʼ?

17. Ящик содержит 10 деталей, среди которых 3 стандартных. Найти вероятность того, что из наудачу отобранных 5 деталей окажется не более одной стандартной.

18. Брошены два одинаковых игральных кубика. Найти вероятность того, что цифра 6 появится хотя бы на одной грани.

19. Для поражения цели достаточно попадания хотя бы одного снаряда. Произведено два залпа из двух орудий. Найти вероятность поражения цели, в случае если вероятность попадания в цель при одном выстреле из первого орудия равна 0,3, а из второго - 0,4.

20. В урне лежит 12 белых и 8 красных шаров. Вынули 8 шаров. Какова вероятность того, что˸ 1) три из них красные; 2) красных шаров вынуто не более трех?

№2. Решить задачу.

1. Монету бросают 5 раз. Найти вероятность того, что ʼʼгербʼʼ выпадает˸ А) менее 2 раз; б) не менее 2 раз.

2. Найти вероятность того, что событие А произойдет не менее 2 раз в 4 независимых испытаниях, в случае если вероятность наступления события А в одном испытании равна 0,6.

3. Событие В произойдет в случае, в случае если событие А наступит не менее 4 раз. Найти вероятность наступления событияВ, в случае если будет произведено 5 независимых испытаний, в каждом из которых вероятность выступления события А равна 0,8.

4. Вероятность наступления события А хотя бы один раз при тех испытаниях равна 0,936. Найти вероятность выступления события А при одном испытании.

5. Вероятность поражения цели хотя бы одной пулей при 4 независимых выстрелах равна 0,59. Какова вероятность поражения цели при одном выстреле?

6. Пусть вероятность того, что наудачу взятая деталь нестандартная, равна 0,1. Найти вероятность того, что среди взятых наудачу 5 деталей не более 2 нестандартных.

7. Пусть вероятность того, что телевизор потребует ремонта в течение гарантийного срока, равна 0,2. Найти вероятность того, что в течение гарантийного срока из 6 телевизоров˸ а) не более одного потребует ремонта; б) хотя бы один не потребует ремонта.

8. Вероятность выиграть по лотерейному билету равна 1/7. Найти вероятность выиграть не менее чем по двум из шести.

9. Вероятность попадания при каждом выстреле равна 0,4. Найти вероятность разрушения объекта͵ если для этого крайне важно не менее трех попаданий, а сделано 15 выстрелов.

10. Найти вероятность того, что в семье, имеющей 6 детей, не менее двух девочек. Предполагается, что вероятности рождения мальчика и девочки одинаковые.

11. Вероятность появления события А при одном испытании равна 0,1. Найти вероятность того, что при трех независимых испытаниях оно появится˸ 1) не менее двух раз; 2) хотя бы один раз.

12. Игральную кость подбрасывают 3 раза. Найти вероятность того, что дважды появится число очков, кратное трем.

13. Событие В появится в случае, в случае если событие А появится не менее четырех раз. Найти вероятность того, что наступит событие В, в случае если будет произведено пять независимых испытаний, в каждом из которых вероятность появления события А равна 0,5.

14. Случайно встреченное лицо может оказаться, с вероятностью p=0,2 брюнетом, с p=0,3 блондином, с p=0,4 шатеном и с p=0,1 рыжим. Какова вероятность того, что среди трех случайно встреченных лиц˸ 1) не менее двух брюнетов; 2) один блондин и два шатена; 3) хотя бы один рыжий?

15. Вероятность хотя бы одного попадания при двух выстрелах равна 0,99. Найти вероятность трех попаданий при четырех выстрелах.

16. В квартире четыре электролампочки. Для каждой лампочки вероятность того, что она останется неисправной в течение года, равна 5/6. Какова вероятность того, что в течение года придется заменить не менее половины лампочек?

17. В ящике имеется по одинаковому числу деталей, изготовленных заводами №1 и №2. Найти вероятность того, что среди пяти на удачу отобранных деталей изготовлены заводом №1˸ 1) две детали; 2) менее двух деталей; 3) более двух деталей.

18.Пусть вероятность того, что телевизор потребует ремонта в течение гарантийного срока из трех телевизоров˸ 1) не более одного потребует ремонта; 2) хотя бы один не потребует ремонта.

19. В ящике лежат несколько тысяч одинаковых предохранителей. Половина из них изготовлена I заводом, остальные - II заводом. Наудачу вынули пять предохранителей. Чему равна вероятность того, что I заводом из них изготовлены˸ 1) два предохранителя; 2) менее двух предохранителей; 3) более двух предохранителей?

20. Отдел технического контроля проверяет изделия на стандартность. Вероятность того, что изделие нестандартно, равна 0,1. Найти вероятность того, что˸ 1) из трех проверенных изделий только одно нестандартное; 2) нестандартным будет только третье по порядку проверенное изделие.

№3. Решить задачу.

1. Вероятность наступления события в каждом из независимых испытаний равна 0,8. Найти вероятность того, что событие наступит 60 раз в 100 испытаниях.

2. Вероятность наступления события в каждом из независимых испытаний равна 0,2. Найти вероятность того, что в 100 испытаниях событие произойдет не менее 20 и не более 30 раз.

3. Вероятность выступления события в каждом из независимых испытаний равна 0,2. Найти вероятность того, что событие произойдет 12 раз в 100 испытаниях.

4. Вероятность рождения мальчика равна 0,51. Найти вероятность того, что среди 100 новорожденных окажется 50 мальчиков.

5. Вероятность поражения мишени при одном выстреле равна 0,8. Найти вероятность того, что при 100 выстрелах мишень будет поражена ровно 75 раз.

6. В опыте Бюффона монета подбрасывалась 4040 раз. При этом ʼʼгербʼʼ выпал 2048 раз. С какой вероятностью можно было ожидать данный результат?

7. Найти вероятность того, что в партии из 800 изделий число изделий высшего сорта заключено между 600 и 700, в случае если вероятность того, что отдельное изделие окажется высшего сорта͵ равна 0,62.

8. Вероятность неточной сборки прибора равна 0,2. Найти вероятность того, что среди 500 приборов окажется от 410 до 430 (включительно) годных.

9. Пусть вероятность того, что покупателю необходима обувь 41-го размера, равна 0,2. Найти вероятность того, что из 70 покупателей не более 120 потребуют обувь этого размера.

10. Всхожесть семян данного растения составляет 90%. Найти вероятность того, что из 800 посеянных семян взойдет не менее 700.

11. Игральную кость подбрасывают 500 раз. Какова вероятность того, что цифра 1 при этом выпадет 50 раз?

12. Вероятность получения по лотерее безвыигрышного билета равна 0,1. Какова вероятность того, что среди 400 наугад купленных билетов не менее 50 и не более 60 безвыигрышных?

13. Чему равна вероятность того, что среди 100 случайных прохожих окажутся 32 женщины (предполагается, что число мужчин и женщин в городе одинаково)?

14. Вероятность наступления события А в каждом из 100 независимых испытаний равна 0,8. Найти вероятность того, что событие А появится в этих испытаниях˸ 1) ровно 90 раз; 2) не менее 80 и не более 90 раз.

15. Вероятность выздоровления больного в результате применения нового способа лечения равна 0,8. Сколько вылечившихся из 100 больных можно ожидать с вероятностью 0,75?

16. Игральную кость подбрасывают 320 раз. Какова вероятность того, что цифра 5 при этом выпадет не менее 70 и не более 83 раз?

17. Вероятность того, что пассажир опоздает к отправлению поезда, равна 0,02. Найти наиболее вероятное число опоздавших из 625 пассажиров и вероятность этого события.

18. При проведении эксперимента монету подбрасывали 4096 раз, причем герб выпал 2068 раз. С какой вероятностью можно было ожидать данный результат?

19. Найти вероятность того, что в партии из 900 изделий число изделий высшего сорта заключено между 600 и 700. Вероятность появления изделия высшего сорта в партии равна 0,8.

Заключение

В настоящее время получило всеобщее признание то, что успех развития многих областей науки и техники существенно зависит от развития многих направлений математики. Математика становится средством решения проблем организации производства, поисков оптимальных решений и, в конечном счете, содействует повышению производительности труда и устойчивому поступательному развитию народного хозяйства.

Использование экстремальных задач при изучении математики оправдано тем, что они с достаточной полнотой закладывают понимание того, как человек ищет, постоянно добивается решения жизненных задач, чтобы получающиеся результаты его деятельности были как можно лучше. Решая задачи указанного типа, наблюдаем, с одной стороны, абстрактный характер математических понятий, а с другой - большую эффективную их применимость к решению жизненных практических задач.

Экстремальные задачи помогают ознакомиться с некоторыми идеями и прикладными методами школьного курса математики, которые часто применяются в трудовой деятельности, в познании окружающей действительности.

Решение экстремальных задач способствует углублению и обогащению наших математических знаний. Через задачи мы знакомимся с экстремальными свойствами изучаемых функций, с некоторыми свойствами неравенств. Эти задачи могут серьезно повлиять на содержание учебного материала, на аспекты применения положений изучаемой теории на практике.

Сегодня в центре внимания - ученик, его личность, неповторимый внутренний мир. Поэтому основная задача - выбрать формы и методы организации учебной деятельности учащихся, которые оптимально соответствуют поставленной цели развития личности. Мастерство учителя состоит в умении сделать содержание своего предмета богатым, глубоким, привлекательным, а способы самостоятельной деятельности учащихся разнообразными, творческими, продуктивными.

Объяснительный текст самостоятельной работы раскрывает новое для учащихся понятие, правило, математический факт. Он заканчивается разъясняющими примерами. Вряд ли одна самостоятельная работа может обеспечить формирование твердых навыков вычислений, преобразований, решений уравнений и т.д. Она и не ставит эту цель. Выполнение упражнений, следующих за объяснительным текстом, должно способствовать сознательному усвоению изучаемой теории. Поэтому в каждую работу включаю разнообразные по своему характеру упражнения.

Решение задач способствует развитию мышления школьников лишь в том случае, если каждый ученик решает задачу сам, прилагая для этого определенные усилия. Поэтому, подбирая задачи, стараюсь чтобы они максимально были приближены к действительности, отражали жизненную ситуацию.

Особое внимание уделяю самостоятельным заданиям, которые формируют умение анализировать, сравнивать, обобщать, выделять главное, контролировать и планировать свою деятельность. Так, при прохождении темы "Решение треугольников", ученикам предлагается домашнее задание: составить рассказ о теоремах синусов и косинусов по плану:

1. Что вы знаете о возникновении теоремы.

2. Какого типа задачи вы можете решать с помощью этих теорем.

3. Как можно использовать эти теоремы в других предметах или в практической жизни человека.

Нередко, с целью развития мышления учащихся, предлагаю ребятам задания по самостоятельному составлению задач. Такие задания могут быть весьма разнообразными. Например, составьте задачу, обратную той, что решена; составьте задачу на такую-то формулу, составьте задачу в стихотворной форме и т.д.

Такие задания систематизируют знания учащихся, учат их видеть основное, повышают речевую активность. Для воспитания познавательной активности школьников использую в своей практике ознакомление их с различными способами доказательства теорем, различными подходами к решению одной и той же задачи.

В перспективе планируется более широко использовать специальные логические упражнения. Для усвоения методов научного познания необходимо учащимся давать задания на применение этих методов, не называя их, например: сравнить (сопоставить или противопоставить), сделать вывод по аналогии, обобщить, конкретизировать, провести классификацию и другое. Благодаря таким упражнениям, представляющим логические задания на программном материале математики, учебная работа школьников превратится в школу логического мышления. При этом будет достигнута цель углубления полученных знаний, интенсивнее будет формироваться интерес, учащихся к изучению школьного курса математики.



Учебное пособие по математике

Список литературы


1. Алгебра и начала анализа: Учебник для 10-11 класса Часть I Учебник. /Под редакцией А.Г. Мордковича - Москва: Мнемозина, 2014.

2. Алгебра и начала анализа: Учебник для 10-11 класса. Часть II Задачник. /Под редакцией А.Г. Мордковича - Москва: Мнемозина, 2014

3. Алгебра и начала анализа: Учебник для 10 класса. Профильный уровень, Часть I Учебник. /Под редакцией А.Г. Мордковича - Москва: Мнемозина, 2014

4. Алгебра и начала анализа: Учебник для 10 класса. Профильный уровень, Часть II Задачник. /Под редакцией А.Г. Мордковича - Москва: Мнемозина, 2014

5. Алгебра и начала анализа: Учебник для 10-11 класса общеобразовательных учреждений /А.Н. Колмогоров, А.М. Абрамов, Ю.П. Дудницын и др.; Под редакцией А.Н. Колмогорова. - Москва: Просвещение, 2012

6. Дидактические материалы по алгебре и началам анализа для 10 класса /Б.М. Ивлев, С.М. Саакян, С.И. Шварцбурд. - Москва: Просвещение, 2012

7. Дидактические материалы по алгебре и началам анализа для 11 класса /Б.М. Ивлев, С.М. Саакян, С.И. Шварцбурд. - Москва: Просвещение, 2012

8. Алгебра: Учебник для 9 класса общеобразовательных учреждений / Ю.Н. Макарычев, Н.Г. Миндюк, К.И. Нешков, С.Б. Суворова; Под редакцией С.А. Теляковского. - Москва: Просвещение, 2010

9. Алгебра и начала анализа: Учебник для 11 класса общеобразовательных учреждений /С.М. Никольский, М.К. Потапов, Н.Н. Решетников, А.В. Шевкин. - Москва.: Просвещение, 2010

10. Алгебра и начала анализа: Учебник для 10 класса общеобразовательных учреждений /С.М. Никольский, М.К. Потапов, Н.Н. Решетников, А.В. Шевкин. - Москва: Просвещение, 2010

11. Алгебра и начала анализа в 9-10 классах: Пособие для учителя /Л.О. Денищева, Ю.П. Дудницын, Б.М. Ивлев и др. - Москва: Просвещение, 2010

12.Алгебра и начала математического анализа: 10-11 классы: учебник для общеобразовательных учреждений: базовый уровень / Ш.А.Алимов, Ю.М.Колягин, М.В.Ткачёва и др. - 16-е изд. - Москва: Просвещение, 2010. - 464 с.

13.Геометрия, 10-11 : учебник для общеобразовательных учреждений: базовый и профильный Уровни / Л.С.Атанасян, В.Ф.Бутузов, С.Б.Кадомцев и др. - Москва.: Просвещение, 2014 - 255с.

14.ЕГЭ 2013. Математика. ЕГЭ. 3000 задач с ответами по математике. Все задания группы В.

Под редакцией Семенова А.Л., Ященко И.В. Москва: Экзамен, 2013 - 544 с.

15.ЕГЭ 2014. Математика. Задачи с параметрами при подготовке к ЕГЭ. Высоцкий В.С. Москва: Экзамен, 2014 - 316 с.

16.ЕГЭ 2013. Математика. 1000 задач с ответами и решениями по математике. Все задания группы С. Сергеев И.Н., Панферов В.С. Москва: Экзамен, 2013 - 304 с.

17.ЕГЭ 2014. Математика. Отличник ЕГЭ. Решение сложных задач. Панферов B.C., Сергеев И.Н. Москва: Интеллект-Центр, 2014. - 92 с.

18.ЕГЭ 2013. Репетитор. Математика. Эффективная методика. Лаппо Л.Д., Попов М.А. Москва: Экзамен, 2013 - 384 с.

19.ЕГЭ 2014. Самое полное издание типовых вариантов заданий ЕГЭ: 2014. Математика. Высоцкий И.Р, Гущин Д.Д, Захаров П.И. и др. Москва: АСТ, Астрель, 2014 - 96 с.


Интернет - ресурсы

1.Электронные образовательные ресурсы нового поколения (ЭОР НП):

2.Федеральный центр ИОР fcior.edu.ru

3.Единая коллекция ЦОР school-collection.edu.ru

4.Единое окно доступа к ОР window.edu.r

5.А.А. Тригонометрические уравнения: методы решений и отбор корней.

down.ctege.info/ege/2012/book/matem/matem2012reshenieC1koryanov.zip

6.ЕГЭ 2012. Математика. Решение типа С4. Планиметрические задачи с неоднозначностью в условии. Корянов А.Г., Прокофьев А.А.

down.ctege.info/ege/2012/book/matem/matem2012-C4prokofev-koryanov.zip

7.Открытый банк задач ЕГЭ: mathege.ru

8.Он-лайн тесты:

uztest.ru/exam?idexam=25

egeru.ru

reshuege.ru/















Учебное пособие по математике

Учебное пособие по математикеУчебное пособие по математике

Учебное пособие по математикеУчебное пособие по математике



математика Страница 103


© 2010-2022